Chimi Sol Fascicule-1 [PDF]

  • 0 0 0
  • Gefällt Ihnen dieses papier und der download? Sie können Ihre eigene PDF-Datei in wenigen Minuten kostenlos online veröffentlichen! Anmelden
Datei wird geladen, bitte warten...
Zitiervorschau

1

UNIVERSITE CHEIKH ANTA DIOP DE DAKAR FACULTE DES SCIENCES ET TECHNIQUES Département de Chimie



COURS DE CHIMIE PHYSIQUE 1ère PARTIE

Année scolaire 2007 – 2008

Mr M. FALL Maître de Conférences

2

Avant Propos

Ce fascicule comprend la première partie du cours de Chimie Physique : rappels sur les concentrations, thermochimie et équilibres chimiques, dispensé aux étudiants de 1ère année de la Faculté des Sciences et Techniques de l’Université Cheikh Anta DIOP et plus particulièrement à ceux de la section PC1. Cette partie du cours est normalement traitée au premier semestre. Nous avons rassemblé dans ce document des séries de TD et des examens proposés (énoncés et corrigés) durant les dernières années scolaires. Toutefois, le fascicule ne saurait remplacer le cours magistral que nous conseillons de suivre régulièrement. Par ailleurs, la présence aux séances de T.D. demeure obligatoire.

3 CHAPITRE I CONCENTRATIONS DES SOLUTIONS 1.1 DEFINITIONS Une solution peut être définie comme un mélange homogène, en proportions variables de deux ou plusieurs substances. Elle comprend :  un milieu dispersif, le solvant, qui est le constituant majoritaire ;  un milieu dispersé, le ou les solutés : ensemble des substances dissoutes dans le solvant. Il existe des solutions dans tous les états de la matière. Sauf indication contraire, il s’agira ici de solutions liquides, et notamment de solutions aqueuses (le solvant est l’eau). 1.2. EXPRESSIONS DE LA COMPOSITION D’UNE SOLUTION Une solution peut être caractérisée par les proportions relatives de ses divers constituants, comme par exemple la quantité de soluté rapportée à une quantité de solution ou de solvant. C’est ainsi que l’on distinguera :  la concentration d’un constituant, qui représente la quantité de substance (masse, nombre de moles, nombre d’équivalents) rapportée à l’unité de volume de solution ;  le titre d’un constituant, qui correspond à la quantité de substance (masse, nombre de moles) rapportée à la masse ou au nombre de moles de solution. 1.2.1 Composition rapportée à un volume 1.2.1.1 Masse volumique et densité d’une solution La masse volumique d’une solution correspond à la masse de solution (ms) par unité de volume de solution (V). Elle est généralement notée ρ.

ρ=

ms V

(g mL-1, g L-1, kg L-1, etc.)

La densité (par rapport à l’eau) d’une solution aqueuse (d) représente le rapport entre la masse volumique (ρ) de cette solution et celle de l’eau prise à 4°C (ρe), soit 1000 g L-1 :

d=

ρ ρe

4 1.2.1.2 Concentration massique (volumique) (ou concentration pondérale) La concentration massique (Cm) représente la masse de soluté (m) par unité de volume de solution (V).

m V

Cm =

(g mL-1, g L-1, kg L-1, etc.).

Attention : Ne pas confondre masse volumique et concentration massique, bien que ces deux grandeurs aient les mêmes unités. 1.2.1.3 Concentration molaire (volumique) La concentration molaire (volumique) (autrefois appelée molarité) est égale au nombre de moles de soluté (n) rapporté à l’unité de volume de solution (V). Elle est notée CM, M, CA ou [A] pour un constituant A donné.

CM =

n V

(mol L-1, M ou molaire).

Exemple : Une solution contenant 0,005 mole d’hydroxyde de calcium par litre de solution est dite 0,005 molaire, 0,005 M ou encore 0,005 mol.L-1. On notera : [Ca2+] = 0,005 M et [OH-] = 0,01 M. Relation entre CM et Cm.

On peut facilement montrer que Cm = M x CM, M étant la masse molaire moléculaire du soluté. Application : Calcul de la concentration massique de la solution de Ca(OH)2 0,005 M. CM = 0,005 M ; M = 40 + 2 x (16+1) = 74 g.mol-1 ; C m = M x CM ; AN : Cm = 74 x 0,005 = 0,37 g L-1. 1.2.1.4 Concentration équivalente

La concentration équivalente (autrefois appelée normalité) est égale au nombre d’équivalentsgrammes de soluté (néq) rapporté à l’unité de volume de solution (V). Elle est notée Céq ou N.

Céq =

néq

(éq-g L-1 N, ou équivalente).

V

La notion d’équivalent-gramme (éq-g) est liée à celle d’échange de particules actives (protons ou électrons) entre deux substances antagonistes : acide-base, oxydant-réducteur. De telles solutions

5 réagissent toujours "équivalent par équivalent" (1 éq-g d’acide avec 1 éq-g de base, 1 éq-g d’oxydant avec 1 éq-g de réducteur et vice-versa), quelles que soient les concentrations molaires respectives et la nature des substances antagonistes. Définition de l’équivalent-gramme

L’équivalent-gramme désigne la quantité (en grammes) de substance mettant en jeu une mole de particules actives. Pour trouver l’équivalent-gramme, il faut écrire la réaction de dissociation acide ou basique, d’oxydation ou de réduction selon le cas, et mettre en évidence le nombre de particules actives mises en jeu (noté p) par mole de substance. Exemples : H+ + Cl- ; p = 1, méq-g = M/1 = 36,5 g ;

- Acides : HCl H2SO4 H3PO4 - Bases : NaOH + H+

2H+ + SO42- ; +

3-

3H + PO4 ;

p = 2, méq-g = M/2 = 49 g; p = 3, méq-g = M/3 = 32,37 g ;

Na+ + H2O ; p = 1, méq-g = M/1 = 40 g ;

Ba(OH)2 + 2H+

Ba2+ + 2H2O ; p = 2, méq-g = M/2 = 85,7 g ;

- Oxydants : KMnO4 + 8 H+ + 5e-

K+ + Mn2+ + 4 H2O ; p = 5, méq-g = M/5 = 31,6 g ;

KMnO4 + 4 H+ + 3e-

K+ + MnO2 + 2 H2O ; p = 3, méq-g = M/3 = 52,7 g ;

K2Cr2O7 + 14 H+ + 6e-

4 Na+ + S4O62- + 2 e- ; p = 1 ; méq-g = M/1 = 158 g ;

- Réducteurs : 2 Na2S2O3 H2O2

2 K+ + 2 Cr3+ + 7 H2O ; p = 6, méq-g = M/6 = 49 g ;

2 H+ + O2 + 2 e- ; méq-g = M/2 = 17 g ;

- Cas des sels : L’équivalent-gramme d’un sel est égal à la masse molaire de ce sel, rapportée au nombre total de charges positives (ou négatives). méq-g (KBr) = 119/1 (p = 1) ; méq-g (Ba(NO3)2) = 216,4/2 (p = 2) ; méq-g (K3PO4) = 212,3/3 (p = 3) ; méq-g (Fe2(SO4)3) = 399,9/6 (p = 6) ; Récapitulatif :

méq =

masse molaire p

p = nombre de protons (acide ou base) ou d’électrons (oxydant ou réducteur) mis en jeu par mole de substance, ou nombre total de charges positives ou négatives dans le cas d’un sel.

6

Céq =

néq V

= p x CM

Application : Calcul de la concentration équivalente de Ca(OH)2 0,005 M Ca(OH)2 + 2 H+

Ca2+ + 2 H2O ;

p = 2 ⇒ Céq = 2 x CM = 2 x 0,005 = 0,01 éq.L-1.

1.2.2 Composition rapportée à une masse ou à un nombre de moles 1.2.2.1 Molalité La molalité d’une solution représente le nombre de moles de soluté par kg de solvant. Nous la noterons

η ; elle s’exprime en mol par kg de solvant (ou molale).

η= n

me (kg)

Remarques :

 Cette grandeur, numériquement très proche de la concentration molaire pour les solutions diluées, présente l’intérêt d’être indépendante de la température, contrairement aux autres expressions de la concentration.

 Bien que rapportée à une masse et non à un volume, la molalité est souvent appelée concentration molale ; elle fait donc exception à la règle distinctive énoncée plus haut.

1.2.2.2 Fraction ou titre massique C’est la masse de constituant rapportée à la masse totale de la solution (ou d’un mélange homogène quelconque). La fraction massique d’un composant i dans un mélange est notée xi.

xi =

mi mi = Σ mi ms Remarque : la somme des fractions massiques de tous les constituants est égale à 1.

1.2.2.3 Pourcentage massique (ou pourcentage pondéral ou titre massique centésimal) C’est la masse de constituant contenue dans 100 g de solution.

%i =

mi Σ mi

x 100

=

mi ms

x 100

= 100 xi

Remarque : la somme des pourcentages massiques de tous les constituants est égale à 100.

7

1.2.2.2 Fraction molaire C’est le nombre de moles de constituant rapporté au nombre total de moles dans la solution. La fraction molaire d’un composant i dans un mélange est notée Xi .

Xi =

ni Σ ni

Remarque : la somme des fractions molaires de tous les constituants est égale à 1.

CAS DES MELANGES BINAIRES Un mélange binaire est un mélange à deux constituants : un solvant et un soluté. Soit un soluté A, dissout dans un solvant (eau) ; les fractions massique (xA) et molaire (XA) sont :

xA =

mA mA + me

; xe = 1 - xA

nA

XA =

nA + ne

; Xe = 1 - XA

En exprimant les masses de solvant et de soluté en fonction de la masse de solution et de la fraction massique du soluté (xA) d’une part, puis en fonction du nombre total de moles et de la fraction molaire du soluté (XA) d’autre part, on obtient les expressions suivantes :

η

=

xA x 1000

η

MA x (1 - xA)

=

XA x 1000 Me x (1 - XA)

D’autres expressions peuvent être aisément établies :

η

=

Cm x 1000 MA x (ρ - Cm)

η

=

CM x 1000 (ρ - MA x CM)

Dans ces relations, Cm (g L-1) et CM (mol L-1) sont respectivement les concentrations massique et molaire de la solution, MA et Me (g mol-1) les masses molaires du soluté et du solvant, et enfin ρ (g L-1) la masse volumique de la solution.

8 CHAPITRE II THERMODYNAMIQUE CHIMIQUE 2.1 RAPPELS SUR LES GAZ 2.1.1 Propriétés générales et caractéristiques des gaz

Les gaz possèdent certaines propriétés, qui les distinguent des liquides et des solides : - faible masse volumique ; - sensibilité à la pression (grande compressibilité) ; - pas de forme fixe, ni de surface ; - mélange toujours homogène (de deux ou plusieurs gaz) ; - libre parcours des molécules dans l’espace. Les paramètres utilisés pour la caractérisation des gaz sont : - la pression, notée P ; elle s’exprime en atmosphères, pascals, bars, etc. - le volume, noté V ; le volume est le plus souvent exprimé en litres. - la température : elle est notée t ou θ si elle exprimée en degrés Celsius (°C) et T si elle exprimée en degrés Kelvin (K) (T et t sont liés par la relation : T = t + 273,15). Ces différents paramètres sont reliés par différentes équations empiriques. On distinguera deux types de gaz : les gaz parfaits et les gaz réels. 2.1.2 Les gaz parfaits

Le modèle du gaz parfait repose sur les hypothèses suivantes : - les molécules sont assimilées à des masses ponctuelles, de volume négligeable devant le volume occupé par le gaz ; - il n’y a pas d’interactions entre les molécules de gaz ; - la distance parcourue par une molécule entre deux collisions est grande par rapport aux dimensions de la molécule ; - les chocs entre molécules (ou contre les parois) sont parfaitement élastiques. Ces hypothèses ne sont jamais vérifiées dans l’absolu, et il n’existe donc pas de gaz réellement parfaits. Toutefois, certains gaz, sous certaines conditions, se comportent plus ou moins comme un gaz parfait : température élevée, pression, masse molaire moléculaire et polarité faibles.

9 2.1.2.1 Lois des gaz parfaits •

Loi de Boyle-Mariotte

T3

A température constante, (transformation

une masse donnée de gaz parfait. La pression P est ainsi inversement propor-

Pression

isotherme) le produit PV reste constant pour

tionnelle au volume V.

T2

T3 > T2 > T1

T1

Volume



Loi de Gay-Lussac P1

A pression constante, (transformation isoba-

masse donnée de gaz parfait. Le volume V est donc proportionnel à la

Volume

re) le rapport V/T reste constant pour une

P3 > P2 > P1 P2 P3

température T absolue. Température (K)



Loi de Charles V1

A volume constant (transformation isochoV3 > V2 > V1

masse donnée de gaz parfait. La pression P est donc proportionnelle à la

Pression

re), le rapport P/T reste constant pour une

V2 V3

température T absolue. Température (K)



Loi générale des gaz parfaits

Les gaz parfaits obéissent simultanément aux trois lois précédentes (Mariotte, Gay-Lussac et Charles), si leurs conditions d’application sont réunies. Dans tous les cas, le comportement d’un gaz parfait peut être décrit par la loi générale des gaz parfaits : Pour n moles de gaz parfaits de volume V, la température T et la pression P sont telles que:

10

PV = nR T

R = constante des gaz parfaits.

Pour connaître la valeur de R, on peut par exemple se placer dans les conditions normales de température et de pression (CNTP) : P0 = 1 atm, T0 = 273,15 K, Vm = 22,414 L.mol-1 ; R = 1 x 22,414 / 273,15 = 0,082 L.atm.mol-1.K-1. Si on utilise les unités du système international, on a : 1 atm = 101325 Pa et donc : R = 101325 x 0,022414 / 273,15 = 8,314 J.mol-1.K-1. 2.1.2.2 Mélange de gaz parfaits

Soit un mélange de gaz parfaits (nombre total de moles = n), dont le volume total est V et la pression totale P. En admettant que le mélange des gaz parfaits se comporte lui-même comme un gaz parfaits, on peut lui appliquer la loi générale : PV = n RT Par ailleurs, chaque constituant du mélange (gaz i) peut être caractérisé par sa pression partielle, son volume réduit ou sa fraction molaire. •

Pression partielle (Pi) :

C’est la pression (fictive) qu’aurait le gaz i s’il occupait seul le volume total (V) du mélange, à la même température. Si ni est le nombre de moles de i, on peut appliquer à ce gaz la loi des gaz parfaits : PiV = ni RT •

(1)

Volume réduit (Vi) :

C’est le volume qu’occuperait le gaz i s’il était à la pression totale P. PVi = ni RT •

(2)

Loi de Dalton :

La pression d’un mélange de gaz parfaits est la somme des pressions partielles des différents gaz constituant ce mélange :

P = Σ (Pi).

En combinant les relations (1) et (2), on peut montrer que :

Pi Vi = P V

=

ni = Xi = Fraction molaire du gaz i n

11 •

Masse molaire moyenne d’un mélange de gaz parfaits

La masse molaire moyenne (ou apparente) d’un mélange de gaz parfaits peut être définie comme étant le rapport de la masse totale de gaz au nombre total de moles de gaz.

M=



m1 + m2 + m3 + ... nt

=

n1 nt

x

M1 +

n2 nt

x

n3

M2 +

nt

x

M3 + ... ==>

M = Σ(Xi Mi)

Densité d’un gaz parfait par rapport à l’air

La densité d’un gaz par rapport à l’air, représente le rapport entre la masse volumique de ce gaz et celle de l’air, dans les mêmes conditions de température et de pression. L’air étant constitué en moyenne de 78 % d’azote, 21 % d’oxygène et 1 % d’argon, on peut montrer aisément que la densité par rapport à l’air d’un gaz quelconque s’exprime par :

d =

M Mair

=

M 29

2.1.3 Les gaz réels Lorsque les pressions deviennent élevées, le comportement du gaz s’écarte de plus en plus de celui du gaz parfait. La relation PV = nRT ne s’applique plus. D’autres équations semi-empiriques seront utilisées.

2.1.3.1 Facteur de compressibilité Le facteur de compressibilité Z est défini

Z = (PVm) / (RT) Vm étant le volume molaire du gaz parfait. Lorsque la pression tend vers 0 (comportement d’un gaz parfait), Z = 1. Le facteur de compressibilité est fonction de la pression (voir figure).

T1

Facteur de compressibilité (Z)

par :

T2

T3

1

Pression

2.1.3.2 Equation de Van Der Waals

12 Pour décrire le comportement d’un gaz parfait, Van Der Waals a proposé l’équation semiempirique suivante :

a n2 (P + ) (V - n b) = n RT V2

b = covolume (volume occupé par les molécules). a = facteur de proportionnalité de l’inter-attraction avec n²/V². n²/V² = pression interne.

2.1.3.3 Equation du Viriel PVm = RT + B P + C P² + D P3 + … B, C, D … sont les 2e, 3e, 4e, … coefficients du Viriel ; ils dépendent de la température.

2.2 LE PREMIER PRINCIPE DE LA THERMODYNAMIQUE 2.2.1 Généralités •

Notion de système

On appelle système la partie de l’univers, constituée par l’objet à étudier. Lorsque l’on parle de système, on isole par la pensée cet objet du reste de l’univers, que l’on appelle milieu extérieur.

Système isolé : pas d’échange de chaleur, ni de matière entre le système et le milieu extérieur. Système fermé : pas d’échange de matière, mais échange de chaleur avec le milieu extérieur. Système ouvert : échange de chaleur et de matière entre le système et le milieu extérieur. •

Enoncé du 1er principe (également appelé principe de conservation de l’énergie) L’énergie d’un système isolé est constante. Cette énergie est notée U et inclut les énergies de translation, rotation, vibration UU = Cste = Cste

des molécules, etc. Elle est caractéristique d’un état donné : c’est une fonction d’état.



Systèmes non isolés : échanges d’énergie

On peut distinguer deux formes d’échanges d’énergie entre un système et le milieu extérieur :

o la chaleur (Q), énergie thermique échangée ; o le travail : énergie mécanique, électrique, chimique, etc.

13 Ces deux formes d’énergie sont équivalentes. Le principe de l’équivalence chaleur-travail découle de l’expérience de Joule : θ (°C)

Une certaine quantité d’énergie mécanique ∆h E méc = W = m g ∆h eau : Eth = Q = m c ∆θ Q = W

est

transformée

en

énergie thermique qui provoque une augmentation de la température de l’eau.

L’égalité entre W et Q exprime donc le fait que le travail et la chaleur sont deux formes d’énergie équivalentes. Pour un système donné, l’énergie interne est la somme des deux formes d’énergie : U = Eméc + Eth. L’énergie interne d’un système ne peut pas être quantifiée dans l’absolu. Toutefois, ses variations peuvent être évaluées. Conséquence : on peut caractériser un système par son énergie interne, et l’évolution du système par la variation de son énergie interne.



Autres énoncés du 1er principe : 

Principe d’équivalence :

Tout apport d’énergie à un système accroît son énergie interne.

∆U = UF – UI = W + Q représente la variation d’énergie interne pour passer d’un état initial I à un état final F.



Principe de l’état initial et de l’état final :

Si plusieurs chemins sont disponibles pour passer d’un état initial I à un état final F, la variation d’énergie interne est indépendante du chemin suivi ; elle ne dépend que de l’état initial et de

l’état final. La variation d’énergie interne est une fonction d’état. Par contre, le travail et la chaleur dépendent du chemin suivi et ne sont donc pas des fonctions d’état.

14 1 >

I

F

∆U = UF – UI = W1 + Q1 = W2 + Q2 = W3 + Q3

>>

2

Convention de signe : Les énergies reçues par le système sont comptées positivement alors que celles cédées par le système sont comptées négativement.

2.2.2 Evaluation des grandeurs énergétiques 2.2.2.1 Lois élémentaires •

Travail élémentaire

Soit l’expérience de compression (ou de détente) réversible d’un gaz enfermé dans un récipient.

dW = F dx = -Pext dV F dx Note : Une transformation est dite réversible lorsqu’elle est réalisable en sens inverse (on dit que la transformation est inversible) et si, dans les deux sens, elle a lieu de façon quasi-stati-

que (c-à-d suffisamment lente pour qu’à chaque instant, on connaisse l’état du système). •

Quantité de chaleur élémentaire

On distinguera ici la quantité de chaleur mise en jeu lors d’une transformation effectuée à volume constant, ou à pression constante. V = Cste : dQ1 ext→syst = d Qv = m x cv x dT = n x CV x dT ; P = Cste : dQ2 ext→syst = d Qp = m x cp x dT = n x CP x dT . cv et cp : capacités calorifiques massiques (J.g-1.K-1) à V et P constants, respectivement. Cv et Cp : Capacités calorifiques molaires (J.mol-1.K-1) à V et P constants, respectivement.

∂Q/∂T)v = m x cv = n x CV •

Energie interne

Expérience de Gay-Lussac :

∂Q/∂T)p = m x cp = n x CP

15

θ (°C) GP vide

On ouvre le robinet, et le gaz parfait (GP) se

calorimètre parfait (système isolé)

détend dans le ballon vide. Le volume du gaz augmente et sa pression diminue. Le thermomètre montre que la température est restée constante durant l’expérience.

Aucun travail n’est effectué et donc δW = 0. De plus ∆θ = 0, et donc δQ = 0. On peut alors en déduire que dU = 0. La pression et le volume n’ont donc pas d’effet sur la variation d’énergie interne d’un gaz parfait. dU = δW + δQ = δQ – PdV. A volume constant, dV = 0, et on obtient : dU = δQv = n CV dT •

Fonction enthalpie

La chaleur échangée lors d’une transformation à volume constant est différente de celle mise en jeu à pression constante. On introduit une nouvelle fonction d’état, l’enthalpie : H = U + PV dH = dU + P dV + V dP = δQ – P dV + P dV + V dP dH = δQ + V dP. A pression constante, dP = 0 et donc : dH = δQP = n CP dT •

Relation entre CP et CV :

La relation entre CP et CV peut être obtenue (de façon simplifiée) en comparant dH et dU : dH = dU + d(PV) . Pour un gaz parfait, PV = nRT et donc : n CP dT = n CV dT + nR dT C P = CV + R

2.2.2.2 Lois intégrales δW = -Pext dV

16 dU = δW + δQ = n CV dT dH = dU + d(PV) = n CP dT Le calcul intégral de ces fonctions ne peut être effectué que si l’on connaît la nature de la transformation. On considère en général que CP et CV sont indépendantes de la température. Dans le cas de transformations réversibles, la pression du gaz est à tout moment égale à la

pression extérieure. (P ext = P) Soit un gaz parfait, subissant une transformation réversible, l’amenant d’un état initial I (PI, VI, TF) à un état final F (PF, VF, TF). La transformation peut être isotherme, isochore, isobare ou adiabadatique. •

Transformation réversible isotherme d’un gaz parfait

Pour une transformation isotherme, la température du gaz reste constante et par conséquent : TF = TI (∆T = 0). Variation d’énergie interne : ∆U = n CV ∆T = 0

∆H = n CP ∆T = 0

Variation d’enthalpie : Travail échangé :



VF



W = - PdV = -nRT VI

VF

dV V

W = - nRT x ln

VF

= - nRT x ln

VI

VI

PI PF

La température est soit donnée, soit calculée à partir des valeurs de PI VI ou PF VF. Chaleur échangée :

Puisque ∆U = 0, Q = -W. Par conséquent, l’expression de la chaleur échangée est : •

Q = nRT x ln

VF

= nRT x ln

VI

Transformation réversible isochore d’un gaz parfait

Pour une telle transformation, le volume reste constant (VF =VI et donc dV = 0).



VF

Travail échangé: W = - PdV = 0 VI

Variation d’énergie interne : ∆U = n CV ∆T = n CV (TF – TI) Variation d’enthalpie :

∆H = n CP ∆T = n CP (TF – TI)

Chaleur échangée :

Q = ∆U = QV = n CV (TF – TI)



Transformation réversible isobare d’un gaz parfait :

PI PF

17 Une transformation isobare se déroule à pression constante (PF = PI = P). Travail échangé:



VF

W = - P dV = - P x (VF - VI) = - nR x (TF - TI) VI

Variation d’énergie interne : ∆U = n CV ∆T = n CV (TF – TI) Variation d’enthalpie :

∆H = n CP ∆T = n CP (TF – TI)

Chaleur échangée :

Q = ∆H = QP = n CP (TF – TI)



Transformation adiabatique d’un gaz parfait:

Dans le cas d’une transformation adiabatique, il n’y a pas d’échange de chaleur entre le système et le milieu extérieur. (Q = 0). Chaleur échangée :

Q=0

Variation d’énergie interne : ∆U = n CV ∆T = n CV (TF – TI) Variation d’enthalpie :

∆H = n CP ∆T = n CP (TF – TI)

Travail échangé :

W = ∆U – Q = ∆U = n CV (TF – TI)



Transformation irréversible

Si la transformation est irréversible, l’expression du travail échangé est toujours:



VF

W = - Pext x dV VI

Si la pression extérieure est constante, on obtient : W = - Pext (VF - VI). Les autres grandeurs peuvent être obtenues en fonction de W et de la nature de la transformation.

2.3. APPLICATION AUX REACTIONS CHIMIQUES Une réaction chimique est une transformation qui modifie la nature chimique des constituants d’un système. Cette transformation s’accompagne d’un effet thermique :



absorption de chaleur : réaction endothermique ;



dégagement de chaleur : réaction exothermique ;



pas d’échanges de chaleur : réaction athermique.

18 Une même réaction peut se produire dans des conditions expérimentales différentes. On distingue généralement deux cas : les réactions effectuées à pression constante et les réactions effectuées à volume constant.

2.3.1 Réaction à volume constant Une telle réaction peut se produire dans une bombe calorimétrique (V constant). QV = Chaleur de réaction à volume constant ; QV = ∆UR = UB – UA La chaleur de réaction à volume constant ne dépend que de l’état initial (avant réaction) et de l’état final (après réaction) ; elle est égale à la variation d’énergie interne de la réaction.

2.3.2 Réaction à pression constante C’est le cas très fréquent des réactions qui s’effectuent à pression atmosphérique constante, alors que le volume du système peut varier. QP = Chaleur de réaction à pression constante ; QP = ∆HR = HB – HA La chaleur de réaction à pression constante ne dépend que de l’état initial (avant réaction) et de l’état final (après réaction) ; elle est égale à la variation d’enthalpie de la réaction. Remarque : Ces résultats sont valables quelles que soient les températures TA et TB. Mais en général (et c’est le cas le plus fréquent), on considère que TA = TB. QV = Chaleur de réaction à température et volume constants ; QP = Chaleur de réaction à température et pression constantes.

2.3.3 Variation d’énergie interne et d’enthalpie dans les réactions chimiques Considérons un système, formé de plusieurs "sous-systèmes", dont chacun ne comprend qu’une seule phase pure. Pour les variables d’état d’un sous-système i donné, on peut écrire : Ui = ni Ui ; Hi = ni Hi ; CPi = ni CP i ; Les variables d’état du système global s’expriment par : U = U1 + U2 + … = Σ(ni Ui) H = H1 + H2 + … = Σ(ni Hi) CP = CP1+ CP2 + …

= Σ(ni CPi).

19 Lors d’une réaction dans un tel système global, les grandeurs U et H vont subir des variations. Pour calculer ces variations, nous considérons le schéma réactionnel général :

υA A + υB B + ….

υC C + υD D + ….

υA, υB, υC et υD sont les coefficients stœchiométriques des constituants A, B, C et D. On peut poser : υi < 0 pour les réactifs et υi > 0 pour les produits. Pour une réaction chimique, on obtient les variations d’énergie interne et d’enthalpie de réaction :

∆UR = -υAUA - υBUB + υCUC + υDUD = ΣυiUi ∆HR = -υAHA - υBHB + υCHC + υDHD = ΣυiHi ∆UR représente la quantité de chaleur à fournir ou à éliminer du système pour que la température du système puisse rester constante lors d’une réaction à volume constant.

∆HR représente la quantité de chaleur à fournir ou à éliminer du système pour que la température du système puisse rester constante lors d’une réaction à pression constante.

2.3.4 Comparaison entre ∆UR et ∆HR : Nous avons précédemment vu (§ 2.2.2.1) que H = U + PV, soit ∆H = ∆U + ∆(PV). Pour une réaction effectuée à pression constante, on a : ∆HR = ∆UR + P (VF – VI). Pour les liquides et les solides peu compressibles, P (VF – VI) est négligeable devant ∆UR. Pour les gaz parfaits, P (VF – VI) = PVF – PVI = PFVF – PIVI = (nF - nI) x RT = RT x ∆n. Finalement :

∆HR = ∆UR + RT ∆n

∆n = variation du nombre de moles de gaz en passant de l’état initial à l’état final. ∆n = nombre total de moles de produits gazeux - nombre total de moles de réactifs gazeux. Exemples: CH4 (g) + 2 O2 (g)

CO2 (g) + 2 H2O (g)

∆n = 1 + 2 – (1 + 2) = 0

CH4 (g) + 2 O2 (g)

CO2 (g) + 2 H2O (l)

∆n = 1 – (1 + 2) = - 2

2.3.5 Loi des additions de chaleur de HESS Comme l’énergie interne (U) et l’enthalpie (H) sont des fonctions d’état, Hess en conclut que leurs variations (∆U et ∆H) doivent être aussi des fonctions d’état, c’est-à-dire indépendantes du chemin suivi. Ce postulat est d’une grande importance, puisqu’il permet de déterminer des chaleurs de réaction qui ne sont pas mesurables directement.

20 La méthode sera illustrée par la détermination de l’enthalpie de la réaction de combustion du carbone en monoxyde de carbone : C (s) + ½ O2 (g)

CO (g)

Quelle est la valeur de ∆HR de cette réaction ? La variation d’enthalpie de cette réaction étant inconnue, on peut la déterminer à partir des variations d’enthalpie des réactions suivantes : (1)

C (s) + O2 (g)

(2)

CO (g) + ½ O2 (g)



CO2 (g)

∆H1 = - 22,5 kJ ;

CO2 (g) ∆H2 = - 16,2 kJ.

1ère méthode : on cherche la combinaison entre les équations des réactions (dont on connaît

les ∆HR), donnant l’équation de la réaction dont on cherche à déterminer le ∆HR de réaction. CO (g) ⇒ ∆HR = ∆H1 - ∆H2

(1) - (2) : C (s) + ½ O2 (g)



2ème méthode : on écrit l’expression de ∆H pour chaque réaction :

C (s) + ½ O2 (g) C (s) + O2 (g)

CO (g) CO2 (g)

CO (g) + ½ O2 (g)

CO2 (g)

∆H = H CO (g) - H C (s) - ½ H O2 (g)

(1)

∆H1 = H CO2 (g) - H C (s) - H O2 (g)

(2)

∆H2 = H CO2 (g) - H CO (g) - ½ H O2 (g)

(3)

En remplaçant H CO (g) et H C (s) par leurs expressions en fonction de ∆H1 et ∆H2, on peut montrer aisément que ∆H = ∆H1 - ∆H2.



3ème méthode : on peut utiliser un cycle :

chemin 1

ETAT INITIAL

ETAT FINAL

1/2 O2 (g) + C (s) + 1/2 O2 (g)

∆H ?

CO (g) + 1/2 O2 (g)

-∆H2

∆H1

CO2 (g) chemin 2 La variation d'enthalpie étant indépendante du chemin suivi (Loi de Hess), on peut écrire : Σ∆H (chemin 1) = Σ∆H (chemin 2), et donc ∆H = ∆H1 - ∆H2.

21 2.3.6 Enthalpie standard (ou enthalpie standard de formation)

La loi de Hess permet de déterminer des chaleurs de réaction à partir des chaleurs d’autres réactions. Toutefois, l’enthalpie et l’énergie interne absolues (H et U) des composés ne sont jamais connues. Seules leurs variations peuvent être déterminées. Puisqu’on mesure toujours des différences d’énergie interne ou d’enthalpie, on peut choisir un état quelconque comme état de référence (ou état standard) et lui attribuer une valeur arbitraire. Par convention internationale, on a défini l’état standard comme l’état d’un élément sous la pression d’une atmosphère et à la température de 25°C (soit T = 298 K), l’élément étant sous

forme de CORPS PUR SIMPLE dans la forme stable correspondant à ces conditions, dites conditions standard. Les enthalpies correspondant à ces états standard sont appelées "enthalpies

standard" et notées ∆H°298. Convention : l’enthalpie standard d’un élément est égale à 0.

Exemple : ∆H°298 (H2) = 0. T > 298 K ⇒ ∆H°298 (H2) > 0 ; T < 298 K ⇒ ∆H°298 (H2) < 0 Le choix de la valeur 0 s’explique par le fait qu’aucune réaction chimique ne permet de transformer un élément en un autre, dans les conditions standard. Enthalpie standard de formation d’un composé :

C’est la variation d’enthalpie accompagnant la formation d’une mole de composé à partir des éléments, dans les conditions standard. Elle est souvent notée ∆H°f et s’exprime en J.mol-1. Exemple : H2 (g) + ½ O2 (g)

H2O (l)

∆H°R = ∆H°f (H2O (l)) = ∆H° 298 (H2O (l)) - ∆H°298 (H2 (g)) – ½ ∆H°298 (O2 (g)). Puisque ∆H°298 (H2 (g)) = ∆H°298 (O2 (g)) = 0 ∆H°f (H2O (l)) = ∆H° 298 (H2O (l)). Finalement, l’enthalpie d’une réaction devient : ∆H°R = Σ∆H°f (produits) - Σ∆H°f (réactifs) = Συi∆H°f (i) Exemple : Calcul de la variation d’enthalpie de la réaction : CH2CH2 (g) + H2 (g)

CH3CH3 (g)

Données : ∆H°f (CH2CH2 (g)) = 52,3 kJ.mol-1, et ∆H°f (CH3CH3 (g)) = -84,6 kJ.mol-1 ∆H°R = ∆H°f (CH3CH3 (g)) - ∆H°f (CH2CH2 (g)) = -84,6 – 52,2 = -136,8 kJ.

22 Remarque : L’enthalpie de réaction doit, en toute rigueur, être exprimée en J (ou en kJ), alors que l’enthalpie de formation est rapportée au nombre de moles de ce composé et s’exprime donc en J.mol-1 (ou en kJ.mol-1). 2.3.7 Energie de liaison Définition : L’énergie d’une liaison covalente représente la variation d’enthalpie

accompagnant la formation d’une mole de liaison dans une molécule gazeuse à partir des atomes pris à l’état gazeux, sous 1 atm et à 298 K. Ainsi, l’énergie de la liaison H-H correspond à la variation d’enthalpie de la réaction : H (g) + H (g)

H2 (g)

Une énergie de liaison est toujours négative puisque l’établissement de la liaison correspond à une stabilité accrue du système, donc à un dégagement de chaleur. L’énergie de dissociation est l’énergie de la réaction inverse : elle est toujours positive (il faut fournir de l’énergie pour rompre une liaison). L’énergie de liaison se calcule à partir de la loi de Hess, si l’on connaît les différentes enthalpies de formation. Exemple : Détermination de l’énergie de la liaison C-H dans le méthane : C (g) + 4 H (g)

CH4 (g)

∆H°f (C (g)) = 714,8 kJ.mol-1 ; ∆H°f (H (g)) = 217,4 kJ.mol-1 ; ∆H°f (CH4 (g)) = - 75,2 kJ.mol-1 ; ∆HR = ∆H°f (CH4 (g)) – ∆H°f (C (g)) – 4 x ∆H°f (H (g)) = -75,2 – 714,8 – 4 x 217,4 = -1659,6 kJ La molécule de CH4 contient 4 liaisons C-H ; par conséquent, la valeur trouvée correspond à la variation d’enthalpie accompagnant la formation de 4 moles de liaison. ∆HR = 4 x E C-H et donc E C-H = ∆HR / 4 = -1659,6/4 = - 414,9 kJ.mol-1. Remarque : la variation d’enthalpie d’une réaction peut être obtenue à partir des énergies de liaison des produits et des réactifs. Exemple : Calculer ∆H de la réaction d’hydrogénation de l’éthylène, à partir des énergies de liaison. En supposant que l’énergie de la liaison C-H dans C2H6 est égale à celle de la liaison C-H dans C2H4, on peut obtenir l’enthalpie standard de réaction en appliquant la loi de Hess :

23

C2H4 (g) + H2 (g) EC=C + 4EC-H

∆HR ?

EH-H

2 C (g) + 4 H (g) + 2 H (g)

C E C-

C2H6 (g)

+6

H E C-

∆ΗR = -EC=C - 4 x EC-H – EH-H + EC-C + 6 x EC-H ; ∆ΗR = EC-C + 2 x EC-H – EH-H - EC=C AN : ∆ΗR = -347,4 + 2 x (-414,9) – (-614,5) – (-434,7) = -128 kJ.

A partir des enthalpies de formation, on obtiendrait : ∆HR = ∆H°f (C2H6 (g)) – ∆H°f (C2H4 (g)) = -84,6 – 52,2 = -136,8 kJ. Cette dernière valeur est légèrement différente de la première. Ceci peut s’expliquer par le fait que les énergies de liaison ont des valeurs moyennes. En outre, le calcul mathématique ne tient pas compte de divers phénomènes comme les équilibres de résonance dans la molécule. 2.3.8 Variation des enthalpies de réaction avec la température – Loi de Kirchhoff

Connaissant l’enthalpie standard ∆H°298 (T0 = 298 K, P = 1 atm) de la réaction : υA A + υB B

υC C + υD D;

Il s’agit de déterminer l’enthalpie ∆H°T de la même réaction, mais à la température T. Pour ce calcul, on passe de l’état initial (réactifs) à l’état final (produits) par deux chemins différents, les deux se déroulant à pression constante :

T0 = 298 K

υA A + υB B ∆HA

T (K)

∆HB

υA A + υB B

υC C + υD D ∆HC

T0 = 298 K

∆HD

υC C + υD D

T (K)

1er chemin :

On effectue la réaction dans les conditions standard (P = 1 atm, T0 = 298 K). La variation d’enthalpie correspondante est ∆H°298. 2ème chemin :



On élève la température des réactifs (υA moles de A et υB moles de B) de T0 à T à pression

constante (1 atm). Soient CA et CB les capacités calorifiques molaires des réactifs A et B, supposées indépendantes de la température. Si aucun changement de phase ne se produit dans cet intervalle de température, on obtient : ∆HA = υACA x (T - 298) ;

∆HB = υBCB x (T - 298).

24 

On effectue ensuite la réaction à la température T (P = 1 atm). La variation d’enthalpie

correspondante est ∆H°T. 

On ramène la température des produits (υC moles de C et υD moles de D) à 298 K (P = 1

atm). Soient CC et CD les capacités calorifiques molaires des produits C et D, supposées indépendantes de la température. ∆HC = υCCC x (298 - T) ;

∆HD = υDCD x (298 - T).

Loi de Hess : ∆H°298 = ∆HA + ∆HB + ∆H°T + ∆HC + ∆HD ∆H°T = ∆H°298 + [(υCCC + υDCD)-(υACA + υBCB)] x (T - 298) ∆H°T = ∆H°298 + [Σ(υprod Cprod) - Σ(υréact Créact)] x (T - 298) ∆H°T = ∆H°298 + [Σ(υi Ci)] x (T-298) On peut généraliser ce résultat et exprimer la variation d’enthalpie d’une réaction à une température T (∆H°T), en fonction de la variation d’enthalpie de la même réaction à une température T0 (∆H°T0) et des capacités calorifiques molaires des produits et réactifs (qui peuvent être fonction de T). On obtient la loi de Kirchhoff : T

∫(Συ C )dT

∆H°T = ∆H°T0 +

T0

i pi

Remarques : 

Si des changements de phase ont lieu entre T0 et T, il faudrait en tenir compte dans

l’expression de la loi de Kirchhoff. Exemple : Soit la réaction H2 (g) + ½ O2 (g)

H2O (l) ;

Donner l’expression littérale de ∆H°398 en fonction de ∆H°298. Entre 298 et 398 K, il se produit une vaporisation de l’eau. Le cycle global est représenté par :

T0 = 298 K

H2 (g) + 1/2 O2 (g)

∆H°298

H2O (l) H

H2O (l)

T = 373 K

−∆Hvap H 3 H2O (g)

T = 373 K

4

∆H1 T = 398 K

∆H2

H2 (g) + 1/2 O2 (g)

∆H°398

H2O (g)

Loi de Hess : ∆H°298 = ∆H1 + ∆H2 + ∆H°398 + ∆H3 - ∆Hvap + ∆H4

T0 = 298 K

T = 398 K

25



373



398

∆H°398 = ∆H°298 + ∆H°vap + Cp H 2O (l) dT + Cp H2O (g) dT -



298

373

398

∫ [C

p H2 (g)

+ 1/2 Cp O2 (g)] dT

298

La loi de Kirchhoff s’exprime, dans le cas d’une réaction effectuée à volume constant, par : T

∆U°T = ∆U°T0 +

∫ T(Συ C )dT i vi

0

26

CHAPITRE III LES EQUILIBRES CHIMIQUES 3.1 GENERALITES - DEFINITIONS •

Réactions totales

Une réaction est dite totale, si au bout d’un temps suffisamment long, le composé en défaut parmi les réactifs a complètement disparu. Exemple : Soit la réaction : H2 (g) + ½ O2 (g)

H2O (g)

En partant de a moles de H2 et b moles de O2, trois cas peuvent être envisagés : b > a/2 : O2 est en excès par rapport à H2 ; a > 2b : H2 est en excès par rapport à O2 ; a = 2b : H2 et O2 sont dans les proportions stœchiométriques. 1er cas :

2ème cas :

H2(g) + ½ O2(g) t0 a b (>a/2) tqcq a-x b–x/2 t∞ 0 b–a/2 •

H2O(g) 0 x a

3ème cas :

H2(g) + ½ O2 (g) t0 a (>2b) b tqcq a–x b–x/2 t∞ a–2b 0

H2O (g) 0 x 2b

H2 (g) + ½ O2 (g) t0 a a/2 tqcq a–x (a–x)/2 t∞ 0 0

H2O (g) 0 x a

Réactions équilibrées

Une réaction est dite équilibrée si, au bout d’un temps suffisamment long, on atteint un état où coexistent tous les réactifs et tous les produits, et pour lequel le système n’évolue plus. Exemple : estérification de l’acide acétique : CH3COOH + C2H5OH

Cette réaction s’effectue en phase

[ester] b) CH3COOC2H5 + H20 t0 1 1 0,67 téq 0,67

CH3COOC2H5 + H2O

CH3C00H + C2H5OH 0 0 0,33 0,33

liquide. En milieu non aqueux, que l’on parte d’une mole d’acide + une mole d’alcool ou d’une mole d’ester + une mole d’eau, les quantités à

0,67 a) CH3COOH + C2H50H t0 1 1 téq 0,33 0,33

CH3C00C2H5 + H20 0 0 0,67 0,67

temps

l’équilibre sont de 0,67 mole d’ester + 0,67 mol d’eau + 0,33 mol d’acide + 0,33 mol d’alcool.

On distingue deux types d’équilibres : les équilibres homogènes et les équilibres hétérogènes.

27

Equilibres homogènes : Un équilibre est dit homogène lorsque tous les constituants sont dans une même phase. Les substances doivent être toutes soit à l’état gazeux, soit à l’état de liquides entièrement miscibles ou dissoutes dans un même solvant. Exemples: H2 (g) + I2 (g)

2 HI (g)

CH3COOH (l) + C2H5OH (l) NH3 (l) + H2O (l)

CH3COOC2H5 (l) + H2O (l)

NH4OH (l)

On dit également que l’équilibre a lieu en phase homogène.

Equilibres hétérogènes Dans ce cas, il y a au moins deux phases distinctes : solide/gaz, solide/liquide, solide/solide, liquide/gaz ou liquide/liquide (dans le cas où les liquides ne sont pas entièrement miscibles). Exemples:

CaO (s) + CO2 (g) H2O (s) C (gr)

CaCO3 (s)

H2O (l) C (diam)

3.2 LOI D’ACTION DE MASSE (LAM) (OU DE GULBERG ET WAAGE) Soit un équilibre du type : aA + bB

cC + dD

La loi d’action de masse permet de relier les caractéristiques (concentration, pression, nombre de moles, etc.) des constituants en exprimant les constantes d’équilibre relatives à ces grandeurs. La loi d’action de masse peut être exprimée uniquement lorsque l’équilibre est atteint (coexistence de tous les constituants, système stationnaire). De plus, la réaction considérée se déroule à

température constante. 3.2.1 Cas des équilibres homogènes •

LAM relative aux concentrations (constante d’équilibre relative aux concentrations). c d [C]éq x [D]éq

KC =

b [A]aéq x [B]éq

Les différentes concentrations sont mesurées à l’équilibre.

28 Exemple: CH3COOH (l) + C2H5OH (l)

KC =

CH3COOC2H5 (l) + H2O (l)

[CH3COOC2H5]éq x [H2O] éq

La concentration de H2O intervient car l’eau n’est pas

[CH3COOH]éq x [C2H5OH]éq

ici un solvant.

La constante KC peut être calculée pour les équilibres en phase homogène gazeuse ou liquide. •

LAM relative aux pressions partielles (constante d’équilibre relative aux pressions partielles) c d PCéq x PDéq

KP =

a b PAéq x PBéq

Les différentes pressions partielles sont mesurées à l’équilibre.

La pression partielle n’ayant pas de sens pour un liquide ou un solide, KP ne peut être exprimé pour les systèmes homogènes non gazeux.

Exemple: H2 (g) + I2 (g) •



a b XA éq x XBéq

PH2 x PI2

Les différentes fractions molaires sont mesurées à l’équilibre.

LAM relative aux nombres de moles (constante d’équilibre relative aux nombres de moles) c

d

a

b

nCéq x nDéq KN =

KP =

LAM relative aux fractions molaires (constante d’équilibre relative aux fractions molaires) c d XCéq x XDéq

KX =

2 HI (g)

2 PHI

nAéq x nBéq

Les différents nombres de moles sont mesurés à l’équilibre.

3.2.2 Relations entre les différentes expressions de la LAM, pour un système homogène gazeux. Pour un système homogène gazeux, où tous les gaz sont parfaits, on peut aisément établir les relations suivantes :

KP = KC (RT)∆n

KP = K X P ∆ n

KN = KC V ∆ n

Dans ces relations, R désigne la constante des gaz parfaits, T la température absolue, V le volume total du mélange à l’équilibre, P la pression totale du mélange à l’équilibre et ∆n la différence

29 entre la somme des nombres de moles de produits et la somme des nombres de moles de réactifs (tous à l’état gazeux).

3.2.3 Cas des équilibres hétérogènes La loi de Gulberg et Waage s’applique uniquement à la phase la plus fluide : phase gazeuse pour un équilibre liquide/gaz ou solide/gaz, et phase liquide pour un système liquide/solide. Exemples: CaCO3 (s)

CaO (s) + CO2 (g):

H2 (g) + ½ O2 (g)

H2O (l) :

KP = PCO2 ;

KC = [CO2],

KP = 1/(PH2 x PO2 0,5 ) ;

etc.

KC = 1/([H2][O2]0,5), etc.

Remarque: Les relations entre les différentes expressions de la LAM restent valables. Toutefois ∆n représente en général, la différence entre les sommes des nombres de moles de gaz dans les produits et dans les réactifs. Exemple: CaCO3 (s)

CaO (s) + CO2 (g): ∆n = 1 + 0 - (0) = 1.

3.2.4 Méthodologie d’étude d’un équilibre chimique •

Notion de coefficient de dissociation

Le coefficient de dissociation d’un réactif (appelé également degré de dissociation, ou encore taux de conversion), noté en général α, représente le rapport entre le nombre de moles dissociées et le nombre de moles initial de ce composé.

α=

nombre de moles dissociées nombre de moles initial

Le nombre de moles de chaque constituant peut être exprimé en fonction du coefficient de dissociation et du nombre de moles du constituant considéré. Exemple : t=0 t qcq

aA + bB nA nB n A - x nB - y

cC+dD

αA = x/nA ; αB = y/nB ; y/x = b/a

Si on part des proportions stœchiométriques de A et B, nA/nB = a/b et par suite, αA = αB. Dans le cas contraire, il est conseillé de raisonner avec le degré de dissociation du réactif en défaut.



Notion de taux (degré) d’avancement d’une réaction

Le taux d’avancement d’une réaction (ξ) peut être défini par rapport à un réactif ou par rapport à un produit :

30 - par rapport à un réactif, ξ correspond au nombre de moles ayant réagi, rapporté au coefficient stœchiométrique du réactif dans la réaction ; - par rapport à un produit, ξ correspond au nombre de moles formées, rapporté au coefficient stœchiométrique du produit dans la réaction. Exemple : aA + bB t=0 nA nB t qcq nA − x nB − (b/a)x

cC + nC nC + (c/a)x

dD nD nB + (d/a)x

ξ = x/a Le taux d’avancement caractérise la réaction elle-même, alors que le coefficient de dissociation caractérise plutôt un réactif donné.



Notion de rendement chimique

Le rendement d’une réaction représente le nombre de moles formées à l’équilibre (pour un produit donné), rapporté au nombre de moles qu’on aurait obtenu pour le même produit si la réaction était totale. Le rendement d’une réaction peut également être défini par rapport à un réactif : dans ce cas, il représente le nombre de moles de réactif ayant disparu, rapporté au nombre de moles de réactif qui auraient réagi si la réaction était totale. Remarque : Le rendement est égal au taux de conversion si au départ les proportions des réactifs sont stœchiométriques.



Exemple d’équilibre à étudier

Considérons la réaction de synthèse de l’ammoniac : N2 (g) + 3 H2 (g) t=0 a b t qcq a – x b – 3x a (1 – α) b – 3aα a (1 - α) b - 3aα nT nT

2 NH3 (g) Σ 0 a+b 2x a + b – 2x (x = nombre de moles de N2 ayant réagi); 2aα a + b – 2aα (α = coefficient de dissociation de N2). 2aα 1 (nT = nombre total de moles de gaz) nT

Si on part des proportions stœchiométriques (b = 3a) t=0 t qcq

N2 (g) a a–x a (1 – α) (1 - α) (4 - 2α)

+ 3 H2 (g) 3a 3a – 3x 3a(1 – α) 3(1 - α) (4 - 2α)

2 NH3 (g) Σ 0 a+b 2x 4a – 2x (x = nombre de moles de N2 ayant réagi); 2aα 4a – 2aα (α = coefficient de dissociation de N2 et H2). 2α 1 (4 - 2α)

31 2

KP =

4α2 (4 - 2α)2

PNH3 3

PN2 x PH2

=

27 (1 - α)4 PT2

3.3 FACTEURS D’INFLUENCE D’UN EQUILIBRE 3.3.1 Enoncé du principe de Le Châtelier (ou principe de modération) Dans un équilibre chimique, toute modification d’un facteur d’équilibre entraîne un déplacement de l’équilibre dans le sens contraire à cette modification. Les facteurs d’équilibre sont des caractéristiques des constituants de l’équilibre (pression, concentration, température, etc.). Ils correspondent aux facteurs qui interviennent habituellement dans la constante d’équilibre.

3.3.2 Influence de la pression totale Une augmentation de la pression déplace l’équilibre dans le sens d’une diminution du nombre total de moles de gaz. Une diminution de la pression déplace l’équilibre dans le sens d’une augmentation du nombre total de moles de gaz.

Exemple :

N2 (g) + 3 H2 (g)

1 2

2 NH3 (g)

Une augmentation de pression déplace l’équilibre dans le sens 1 alors que le sens 2 est favorisé par les plus faibles pressions.

3.3.3 Influence d’un gaz inerte L’introduction d’un gaz inerte à température constante a pour effet d’augmenter le nombre total de moles de gaz. La pression partielle d’un constituant i est Pi = ni RT / V.

∗ Si l’introduction du gaz inerte a lieu à volume total constant, il n’y aura pas de modification des pressions partielles des constituants de l’équilibre, et donc pas de la valeur de KP. Dans ce cas, il n’y a pas de déplacement de l’équilibre.

∗ Si par contre l’introduction d’un gaz inerte a lieu à pression totale constante, on a une augmentation de volume du système, donc une diminution des pressions partielles des constituants. Le système évolue dans le sens d’une augmentation du nombre de moles de gaz.

3.3.4 Influence de la température 3.3.4.1 Prévisions qualitatives

32 L’augmentation de la température (qui équivaut à un apport de chaleur) déplace l’équilibre dans le sens endothermique, alors que la diminution de la température entraîne un déplacement de l’équilibre dans le sens exothermique. 1

Exemple : N2 (g) + 3 H2 (g)

2 NH3 (g)

2

∆H = -92,5 kJ

La synthèse de l’ammoniac est favorisée par les plus basses températures, alors que la décomposition de NH3 (sens 2) est favorisée par les plus hautes températures. Remarque : Le principe de Le Châtelier permet de faire des prévisions qualitatives sur les aspects thermodynamiques d’une réaction (par exemple le rendement). Un déplacement de l’équilibre donc être compris comme une variation du rendement de la réaction, compte

non tenu des aspects cinétiques. 3.3.4.2 Variation de la constante d’équilibre avec la température : Equation de Van’t Hoff Les constantes d’équilibre (KC et KP) varient avec la température suivant les relations : d (ln KC) dT

=

∆U RT2

d (ln KP) dT

et

=

∆H RT2

Ces relations sont appelées équations de Van’t Hoff. Leur démonstration est hors-programme. Conséquences : ∆H > 0 ⇒ KP augmente avec l’augmentation de la température ; ∆H < 0 ⇒ KP diminue avec l’augmentation de la température. L’équation de Van’t Hoff confirme ainsi les prévisions qualitatives que l’on a pu faire quant à l’effet de la température sur le déplacement d’un équilibre (principe de Le Châtelier).

Exemple : N2 (g) + 3 H2 (g)

1 2

2 NH3 (g)

∆H = < 0

Le calcul des valeurs de KP à 298 et 673 K donne les résultats suivants : KP (298 K) = 5,52 x 104 atm-2, et KP (673 K) = 2 x 10-4 atm-2. La synthèse de l’ammoniac est donc favorisée par les plus basses températures ; elle est exothermique. Application : Soit un domaine de température (T1↔T2) suffisamment petit pour qu’on puisse considérer que ∆H est constant. On a, par intégration entre T1 et T2 de l’équation de Van’t Hoff :

ln

KP 2 KP 1

=-

∆H 1 R

T2

1 T1

Exemple: Calcul de ∆H de la réaction:

N2 (g) + 3 H2 (g)

2 NH3 (g)

33 T1 = 298 K ⇒ KP1 = 5,52 x 104 atm-2; T2 = 673 K ⇒ KP2 = 2 x 10-4 atm-2. ∆H = R x T1T2 x ln (KP2/KP1) / (T1 - T2) A.N: ∆H = 8,31 x 298 x 673 / (673 - 298) x ln (2 x 10-4 / 5,52 x 104) = -86,4 kJ.

3.3.5 Variance Nous avons vu précédemment (§ 3.3.1) que les facteurs d’équilibre correspondent à l’ensemble des paramètres qui peuvent affecter l’équilibre : la température, la pression totale, les pressions partielles (ou les concentrations). Problème : Quel est le nombre de facteurs d’équilibre que l’on peut fixer arbitrairement sans modifier l’état d’équilibre Ce nombre est appelé degré de liberté ou variance. •

Définition de la variance

La variance représente le nombre de facteurs indépendants d’un équilibre. •

Calcul de la variance

On applique la règle des phases : V = n – r + p - ϕ, avec : V = variance ; n = nombre de constituants ; r = nombre de relations liant les caractéristiques des constituants ; Il y a toujours au moins une relation par équilibre (l’expression de la constante d’équilibre). On doit tenir compte, le cas échéant, des relations particulières qui dépendent des données de l’équilibre. p = nombre de facteurs physiques qui interviennent dans l’équilibre. En général p = 2 (pression et température) ; Si ∆H = 0, la température n’est pas facteur d’équilibre ; Si ∆n = 0, la pression n’est pas facteur d’équilibre. ϕ = nombre de phases. Exemple 1: H2 (g) + I2 (g)

2 HI (g)

S’il n’y a pas de conditions particulières : n = 3 (les constituants sont H2, I2 et HI) ; r = 1 ; p = 1, car ∆n = 0 et donc la pression n’est pas facteur d’équilibre ; ϕ = 1 (une seule phase gazeuse) ; V = n - r + p - ϕ = 3 - 1 + 1 - 1 = 2 (système bivariant).

34 Si on part de HI qu’on chauffe, il y a une relation supplémentaire (PH2 = PI2). V = n - r + p - ϕ = 3 - 2 + 1 - 1 = 1 (système monovariant). Exemple 2: H2O (g) + Fe (s)

FeO (s) + H2 (g)

n = 4 (les constituants sont H2O, Fe, FeO et H2) ; r=1; p = 1, car ∆n = 0 et donc la pression n’est pas facteur d_équilibre ; ϕ = 3 (une phase gazeuse, deux phases solides) ; V = n - r + p - ϕ = 4 - 1 + 1 - 3 = 1 (système monovariant). Exemple 3: AgCl (s)

Ag+aq + Cl-aq

n = 3 (les constituants sont AgCl, Ag+ et Cl-) ; r=1; p = 1, car ∆n = 0 et donc la pression n’est pas facteur d’équilibre. ϕ = 2 (une phase solide, une phase liquide) ; V = n – r + p - ϕ = 3 – 1 + 1 – 2 = 1 (système monovariant). Exemple 4: CaCO3 (s)

CaO (s) + CO2 (g)

n = 3 (les constituants sont CaCO3, CaO et CO2) ; r=1; p=2; ϕ = 3 (deux phases solides, une phase gazeuse) ; V = n – r + p - ϕ = 3 – 1 + 2 – 3 = 1 (système monovariant). Exemple 5:

CaCO3 (s) CO2 (g)

CaO (s) + CO2 (g) CO (g) + ½ O2 (g)

On part de CaCO3 (s) qu’on chauffe; n = 5 (les constituants sont CaCO3, CaO, CO2, CO et O2) ; r = 3 (Kp1, Kp2 et PCO = 2PO2) ; p=2; ϕ = 3 (deux phases solides, une phase gazeuse) ; V = n – r + p - ϕ = 5 – 3 + 2 – 3 = 1 (système monovariant).

3.3.6 Règle des densités Si une masse constante de gaz parfait subit une transformation l’amenant d’un état (1) à un état (2), le rapport des densités est l’inverse du rapport des nombres de moles. Soient m1 la masse de gaz à t1 et d1 sa densité, m2 la masse de gaz à t2 et d2 sa densité ;

35 m1 = n1 x M1 et m2 = n2 x M2 m1 = m2 ⇔ n1 / n2 = M2 / M1 = d2 / d1

Application aux réactions chimiques :

Exemple 1: N2 (g) + 3 H2 (g) t1 n0 3n0 t2 n0 x (1-α) 3n0 x (1-α) m1 = 28 x n0 + 2 x 3n0 = 34 n0 ;

2 NH3 (g) 0 4n0 = n1 2n0α 4n0-2n0α = n2

m2 = 28 x n0 (1 - α) + 2 x 3n0 (1 - α) + 17 x 2n0α = 34 n0.

On voit bien que la masse de gaz reste constante au cours de la réaction.

Vérification de la règle des densités : n1/n2 = 4/(4 - 2α) ⇒ n1 / n2 = 2/(2 - α). Masse molaire moyenne du mélange à l’équilibre:

M2 = XN2 x M N 2 + XH2 x MH 2 + XNH3 x MNH3 =

28 (1 - α) + 2 x 3 (1 - α) + 17 x 2 α 4-2 α

Masse molaire moyenne du mélange à t1 : M1 = (XN2 x MN 2 + XH2 x MH2) = (28 x 1 + 2 x 3)/4 = 8,5 g mol-1.

d2 / d1 = M2 / M1 = 34/(4 - 2 α) / 8,5 ==> d2 / d1 = 2/(2 - α). La règle des densités est donc bien applicable pour ce cas. Exemple 2 : H2O (g) + Fe (s) t1 n0 / t2 n0 x (1-α) / m1 = 18 x n0; m2 = 18 x n0(1 - α) + 2 x n0α = 18 n0 - 16 n0α. m1 ≠ m2 ⇒ la règle des densités ne s’applique pas.

FeO (s) + H2 (g) / 0 / n0 α

Σ n0 = n1 n0 = n2

=

34 4-2 a

36

EXERCICES ET PROBLEMES Enoncés des TD, partiels et examens proposés durant les années 2001-2002 et 2002-2003 A) Concentrations des solutions A-1) Série de TD n° 1 (2001-2002) Exo 1: 15 cm3 d’une solution aqueuse A de carbonate de sodium anhydre contiennent 1,65 g de soluté. La masse volumique de la solution est ρ = 1170 kg m-3. Calculer : a) La concentration massique ; b) La concentration molaire volumique ; c) La fraction molaire et massique ; d) Le pourcentage massique ; e) La concentration équivalente ainsi que sa molalité.

On donne : Na = 23 ; C = 12 ; 0 = 16. Exo 2: Une solution commerciale d’acide nitrique (HNO3, solution A) contient 78,07 % en masse de cet acide ; la densité est d = 1,454. On demande : a) La masse d’acide contenue dans un litre de A ; b) La molalité en acide nitrique ; c) La concentration molaire volumique en acide nitrique ; d) Le volume d’eau à ajouter à 55,52 mL de la solution A pour obtenir une solution B de concentration équivalente en acide nitrique égale à 0,5 N ; e) La densité de la solution B par rapport à l’eau.

Données : N = 14 ; O = 16 ; H = 1. Exo 3: Calculer la masse volumique de la solution obtenue en mélangeant 2 volumes de pentane (ρ = 0,6263 g mL-1) et 5 volumes d’hexane (ρ = 0,6638 g mL-1), sachant qu’il se forme 7 volumes de solution.

Exo 4:

37 Calculer les masses de solutions de CaCl2 1,001 molale et 6,006 molale à mélanger pour obtenir 240 g de solution de CaCl2 3,861 molale de masse volumique 1,2816 gmL-1. Quelle est la concentration équivalente de la solution finale ?

Exo 5: Déterminer la masse de KOH 20,16 % nécessaire pour neutraliser ces solutions aqueuses : a) 200 mL H3PO4 5,39 molaire; b) 125 mL H2SO4 3,92 N; c) 150 g HCl 8,03 molale ; d) 75 mL HNO3 de concentration massique volumique 0,126 g mL-1; e) 239 g d’acide formique de fraction molaire 0,04 ; f) 50 mL KHCO3 10 %, ρ = 1,0674 g mL-1;

Exo 6: Rappel: eau de Javel (NaClO) : La solution d’eau de Javel ou hypochlorite de sodium est une solution oxydante et son titre commercial exprimé en degré chlorométrique représente le volume (en litres) de chlore dégagé par un litre de solution de NaClO conformément à la réaction : NaClO + 2 HCl

NaCl + Cl2 + H2O

Rappel: eau oxygénée (H2O2) : L’eau oxygénée peut réagir comme un oxydant : ou comme un réducteur : H2O2

H2O2 + 2 H+ + 2 e-

2 H+ + O2 + 2 e-

2 H2O

(1)

(2)

Le titre d’une solution d’eau oxygénée, dans le commerce, est exprimé, en volume et représente le volume (en litres) d’oxygène dégagé par un litre de solution suivant la réaction de décomposition de l’eau oxygénée : H2O2

1/2 O2 + H2O

6-A: Analyse d’une eau oxygénée : Une eau oxygénée est analysée par titrage manganimétrique comme suit : pour 20 cm3 de solution primitive, on a utilisé 40 cm3 de solution titrée à 31,6 g L-1 de permanganate de potassium. Quel est le titre en volume de cette eau oxygénée ?

6-B: Analyse d’une eau de Javel : Une solution d’eau de Javel contient 1,49 g de NaClO dans 200 mL de solution Calculer le titre chlorométrique de la solution, ses concentrations équivalente et molaire, ainsi que le volume d’eau à ajouter à 200 mL de cette solution pour obtenir une solution 1,8°.

38

A-2) Série de TD n° 1 (2002-2003) Exo 1: On dissout 20,5 g de H2SO4 (pur) dans 0,5 litre d’eau, la solution obtenue a une masse volumique de 1,025 kg L-1. Calculer : a) la concentration massique b) La concentration molaire volumique c) La concentration équivalente d) La fraction molaire et la fraction massique de chaque constituant e) La molalité f) La densité.

Donnée : La masse molaire de H2SO4 = 98 g mol-1. Exo 2: Un flacon d’acide orthophosphorique porte les indications suivantes: M = 98; d = 1,71; % = 85 %. Calculer les concentrations équivalente et molaire de la solution. A partir de cette solution on veut réaliser 100 litres d’une solution d’acide phosphorique 2,97 M. Quels volumes d’acide et d’eau faut-il mélanger pour préparer cette solution. Quel est le pourcentage en masse d’acide dans la nouvelle solution.

Exo 3: On mélange 220 mL d’une solution A de Na2CO3 à 40 % (d = 1,824) avec 522 g d’une solution B de Na2CO3 à 16,17 éq-g L-1 (d = 1,775) et 515,6 mL d’une solution C de Na2CO3 4 molales (d = 1,173). Calculer : a) La concentration molaire des solutions A, B et C. b) La concentration molaire de la solution finale.

Exo 4: Une solution aqueuse est 2,02 molale en H2S04, 1,23 molale en HNO3 et 10 % en HCl. Déterminer les fractions molaires des trois acides.

Exo 5: Soient une solution A d’acide sulfurique à 90 % et une solution B d’acide sulfurique 4 molales, de masse volumique ρ = 1,205 g mL-1. a) Quelle est la masse de la solution A qu’il faut mélanger à 1,5 L de la solution B pour obtenir 2 L d’une solution C de H2SO4 de concentration équivalente égale à 6,4 éq-g L-1 ?

39 b) Quelle est la masse de la solution C. c) Quel volume de base (NaOH 3,2 mol L-1) faudra-t-il verser pour neutraliser 15 mL de la solution C ? d) 30 mL d’une solution de Na2CO3 permet de neutraliser 15 mL de la solution C. Calculer la concentration massique de Na2CO3.

Exo 6: On désire réaliser en milieu acide une solution de permanganate de potassium pour doser une solution de sulfate ferreux. Ecrire la réaction d’oxydo-réduction. a) Quelle masse de permanganate sec faut-il peser pour réaliser une solution 1 éq-g L-1 ? b) Quelle masse de sulfate ferreux hydraté (FeSO4,7H2O) faut-il pour réaliser 0,5 L de solution de sulfate ferreux permettant de neutraliser 0,5 L de la solution de KMnO4.

On donne : Mn = 55 ; H = 1 ; O = 16 ; K = 39 ; Fe = 56 ; S = 32. Exo 7 : Rappel: Eau de Javel ( NaClO ) La solution d’eau de Javel ou hypochlorite de sodium est une solution oxydante et son titre commercial exprimé en degré chlorométrique représente le volume de chlore (en litres, mesuré dans les CNTP) dégagé par un litre de solution de NaClO conformément à la réaction : Cl2 + NaCl + H2O NaClO + 2 HCl Rappel: Eau oxygénée (H2O2 ) L’eau oxygénée peut réagir comme un oxydant : Ou comme un réducteur :

H2O2 + 2 H+ + 2e-

2 H+ + O2 + 2e- (2)

H2O2 2 H2O

(1)

Le titre d’une solution d’eau oxygénée est exprimé dans le commerce, en volume et représente le volume d’oxygène (en litres, mesuré dans les CNTP) dégagé par un litre de solution suivant la réaction de décomposition de l’eau oxygénée : H2O2

1/2 O2 + H2O

7-A: Sur un flacon contenant de l’eau de Javel il est marqué 8°. Que signifie cette information ? En déduire les concentrations molaire, équivalente et massique de cette solution. Calculer le volume à prélever de ce flacon pour préparer 1 litre de solution 0,1 mol L-1.

7-B: Une solution A de 250 mL contient 0,068 g de H2O2. On ajoute à cette solution VB (mL) d’une solution (B) décinormale de H2O2. Calculer VB si le titre de la solution finale est de 0,112 volume.

40

B) Gaz, thermodynamique chimique B-1) Série de TD n° 2 (2001-2002) Exo 1: A partir de l’équation d’état d’un gaz parfait, calculer les valeurs de R en fonction des dimensions de P et V.

Exo 2: Le gaz à l’eau a la composition suivante (pourcentage en masse). H2

CO

N2

CO2

CH4

6,43

67,82

10,71

14,02

1,02

Exprimer le pourcentage de la composition du gaz en volume, moles et en fractions molaires et calculer la masse volumique du mélange gazeux à la température de 400°C et la pression de 1,5 atm, en supposant que le mélange gazeux se comporte comme un gaz parfait dans ces conditions.

Exo 3: Une masse égale à 2 g d’un gaz A est introduite dans un ballon dans lequel on a préalablement fait le vide. La température étant constante et égale à 25°C, la pression mesurée est de 1 atm. Si on ajoute aux 2 g de gaz A de masse molaire M1, 3 g d’un gaz B de masse molaire M2, la nouvelle valeur de la pression est de 1,5 atm. Assimilant A et B à des gaz parfaits, calculer le rapport de leur masse molaire M2/M1.

Exo 4: Un ballon de verre vide pèse 37,9365 g. Rempli d’air (M = 29) sous une atmosphère et à 25°C, il pèse 38,0739 g. Rempli d’un mélange d’éthane (C2H6) et de méthane (CH4) il pèse 38,0347 g. 1°) Calculer la fraction molaire du méthane dans le mélange gazeux 2°) On réalise à partir de 25°C, la compression isochore d’une mole de ce mélange gazeux en apportant une chaleur de 60 Joules. 2-a) Calculer la variation de l’énergie interne du mélange. 2-b) Calculer la température à la fin de la compression.

On donne : CH4 : CP = 35 J mol-1 K-1 - C2H6 : CP = 50 J mol-1 K-1. Exo 5:

41 Pour un certain gaz parfait, les chaleurs molaires à volume constant et à pression constante valent respectivement 5/2 R et 7/2 R. Une mole de ce gaz est à la pression P1 et occupe un volume V1 à la température T1. On lui fait subir une série de transformations quasi-statiques : - A→ B : chauffage à pression constante jusqu’au volume 2 V1 ; - B → C : compression isotherme jusqu’au volume V1 ; - C→ A : refroidissement à volume constant jusqu’à l’état initial. Représenter cette série de transformations dans un diagramme (P,V) et calculer, pour chacune d’elles, les quantités de travail et de chaleur reçues par le gaz.

Exo 6: Connaissant les valeurs de ∆H à 25°C et 1 atm correspondant aux réactions suivantes : 2 CO2 (g) + 3H2O (l)

∆H1 = -372,8 kcal

(1)

C2H6 (g) + 7/2 O2 (g)

(2)

H2 (g) + 1/2 O2 (g)

(3)

2 C (graphite) + 3 H2 (g)

C2H6 (g)

∆H3 = -20,2 kcal

(4)

2 C (graphite) + 2 H2 (g)

C2H4 (g)

∆H4 = +12,6 kcal

∆H2 = -68,3 kcal

H2O (l)

Calculer : 1°) ∆U lors de la réaction d’hydrogénation de l’éthylène à 25°C selon l’équation : C2H4 (g) + H2

C2H6 (g)

2°) L’enthalpie de combustion de l’éthylène à 25°C.

Exo 7: Enthalpie standard de combustion des alcanes. Estimer, pour les alcanes linéaires de CH4 à C5H12 (g), l’enthalpie standard de combustion à l’aide des données suivantes :

∆H°sub (C(s)) = + 717 kJ.mol-1; ∆H°f (CO2 (g)) = -393,51 kJ.mol-1; ∆H°f (H2O (1)) = -285,85 kJ.mol-1; Liaison

C–C

C–H

H–H

∆H°dis (kJ.mol-1)

348

414

436

Exo 8: Soit la réaction : 4 HCl (g) + O2 (g)

2 H2O (g) + 2 Cl2 (g)

1°) Calculer l’enthalpie standard de réaction à 298 K, puis à 723 K. 2°) En déduire l’énergie interne standard de réaction à 723 K.

42 3°) On suppose que la réaction se déroule dans un réacteur fermé adiabatique en introduisant initialement à 723 K, 4 moles de chlorure d’hydrogène, 2 moles de dioxygène et 8 moles de diazote. Quelle est la température maximale atteinte au cours de la réaction ?

Données à 298 K:

HCl (g)

O2 (g)

N2 (g)

H2O (g)

Cl2 (g)

∆H°f (kJmol-1)

-92,3

0

0

-241,8

0

C°P (JK-1mol-1)

30,1

31,4

30,2

37,6

38,4

Espèces

Exo 9: Dans une bombe calorimétrique, on réalise avec un excès d’oxygène sous pression la combustion complète de 0,100 g d’acide benzoïque. Sachant que la combustion de 1 g d’acide libère 6300 calories et que la différence de température observée est de 0,90°C, on demande de déterminer la "masse en eau" du calorimètre. Dans une deuxième expérience, on introduit sous forme gazeuse 0,440 g de propane normal (C3H8). La différence de température observée est de 8,00 degrés. 1°) Calculer l’enthalpie de combustion molaire (l’eau formée est sous forme liquide). 2°) La combustion a été amorcée par un fil de fer de 0,190 g et la combustion du fer libère 1572 calories par gramme de substance. Doit-on effectuer une "correction" sur la valeur calculée plus haut ? Si oui, quelle est la nouvelle valeur corrigée ?

B-2) Série de TD n° 2 (2002-2003) Exo 1: On considère 10 g d’oxygène dans les conditions normales A (P0 = 1 atm, T0 = 273 K, Vo) : 1°) Calculer le nombre de moles d’oxygène et la valeur du produit P0Vo. 2°) On effectue une transformation isochore AB qui double la pression. Calculer TB. 3°) On effectue une transformation isobare AC qui double le volume. Calculer TC. 4°) Calculer les variations d’énergie interne ∆UAC et ∆UAB et d’enthalpie ∆HAC et ∆HAB. 5°) On considère l’adiabatique réversible qui passe par le point C. La température du point D de cette adiabatique qui correspond au volume initial est 721 K. Calculer le travail échangé durant le cycle BDCB.

Données : CV = 5/2 R; CP = 7/2 R Exo 2: Calculer l’enthalpie standard de formation de PbO2 (s) connaissant les enthalpies standard des réactions suivantes à 25°C.

43 Pb (s) + ½ O2 (g) → PbO (s)

∆H1 = -219 kJ;

Pb3O4 (s) → 3 PbO (s) + ½ O2 (g)

∆H2 = +76,9 kJ;

Pb3O4 (s) + O2 (g) → 3 PbO2 (g)

∆H3 = -94,9 kJ.

Exo 3: Calculer la variation d’enthalpie ∆H de la réaction :

Cdiamant → Cgraphite

Sachant que les enthalpies de combustion du diamant et du graphite sont respectivement égales à –94,48 et –94,03 kcal mol-1.

Exo 4: Calculer la chaleur spécifique d’une substance, sachant que 112 calories sont nécessaires pour augmenter de 10 degrés la température d’un échantillon de 100 g.

Exo 5: a) Calculer l’enthalpie standard de formation de la pyridine, en phase gazeuse à 298 K, à partir des données ci-dessus : C5H5N (l) + 25/4 O2 (g)

5CO2 (g) + 5/2 H2O (l) + 1/2 N2 (g)

∆HR = -2779,5 kJ mol-1.

∆H0f CO2 (g) = -393,1 kJ mol-1; ∆H0f H2O (l)= -285,6 kJ mol-1; ∆H0vap C5H5N = 40,4 kJ mol-1; H

H

b) Calculer cette même grandeur en supposant que la pyri-dine est

C C

représentée par la formule développée ci-contre:

H C

N C

c) Comparer les résultats obtenus en a) et b) et conclure.

C

H

Données : Energies de liaison : C – H : C – C (kJ mol) -413,0 - 347,3

H

C=C

C–N

C=N

H–H

N–H

N≡N

- 614,4

- 291,3

- 614,4

- 435,5

- 390,4

-945,5

Enthalpie de sublimation du carbone: ∆H0sub C (s) = 716,7 kJ mol-1;

Exo 6: L’enthalpie standard de formation de l’eau à 25°C est égale à –285,6 kJ mol-1. Calculer la valeur de cette enthalpie à 400°C.

Données : Chaleur latente de vaporisation de l’eau à 100°C : Lvap : 40,7 kJ mol-1; Capacités calorifiques : Composé

H2(g)

O2

CP (J K-1mol-1) 29,97 + 4,18.10-3T 28,26 + 2,53.10-3T

H2O(l)

H2O(g)

75,47

30,01 + 10,71.10-3T

44

Exo 7: 125 cm3 d’une solution N/2 de soude sont mélangés dans un calorimètre à 125 cm3 d’une solution N/2 d’acide chlorhydrique, les deux solutions et le calorimètre étant à 14°C au début de l’expérience. En admettant que les solutions ont la même chaleur spécifique et la même densité que l’eau et que le calorimètre est équivalent à 10 g d’eau, calculer la chaleur de neutralisation de n’importe quel acide fort par n’importe quelle base forte, rapportée à une mole, sachant que la température de l’ensemble atteint 17,3°C.

C) Equilibres chimiques C-1) Série de TD n° 3 (2001-2002) Exo 1: Sous l’action de la chaleur, le pentachlorure de phosphore se dissocie comme suit : PCl5 (g)

PCl3 (g) + Cl2 (g)

A 230°C et sous une atmosphère, la densité par rapport à l’air du mélange gazeux est de 4,62. On demande : 1°) Le degré de dissociation de PCl5 dans ces conditions de température et de pression. 2°) La constante d’équilibre relative à la pression (KP) 3°) Le degré de dissociation à la même température, mais sous une pression de 0,2 atm ; montrer que le résultat pourrait être prévu qualitativement.

Exo 2: A 0°C et sous 1 atm, la constante d’équilibre de la réaction :

N2O4 (g)

2 NO2 (g)

est KP = 0,05 atm. a) Calculer la variation du degré de dissociation lorsque la pression diminue de moitié, la température restant constante. b) Toujours à 0°C, calculer la pression minimale nécessaire pour obtenir un degré de dissociation au plus égal à 0,01.

Exo 3: A 2000 K, la constante d’équilibre de dissociation du dioxyde de carbone : CO2 (g)

CO (g) + ½ O2 (g)

est KP = 1,45.10-3 atm½. Donner à cette température la composition de la phase gazeuse obtenue à l’équilibre sous une pression totale de 1 atm à partir du CO2 seul et calculer KC.

Exo 4:

45 Lorsqu’on chauffe du chlorure de sulfuryle SO2Cl2, il se volatilise complètement à 70°C, puis se dissocie partiellement en chlore et anhydre sulfureux. 1°) Dans une première expérience, 1,95 g de ce composé est chauffé à θ = 157°C ; on constate que la vapeur formée occupe un volume de 1 litre sous la pression de 1 atmosphère. 1-a) Quel est le degré de dissociation α du chlorure de sulfuryle à 157°C 1-b) Calculer la constante de dissociation KP de ce produit à cette température. 2°) Dans une seconde expérience, on ajoute 1,95 g de chlorure de sulfuryle liquide à 1 litre de chlore à la température de 18°C, contenu dans une enceinte fermée sous la pression de 1 atmosphère ; on chauffe le tout à la température de 157°C. On demande : 2-a) le degré de dissociation α' du chlorure de sulfuryle dans ces conditions ; 2-b) la pression totale développée dans l’enceinte ; 2-c) Comparer α et α' et expliquer.

Exo 5: On considère l’équilibre en phase gazeuse :

N2 (g) + 3 H2 (g)

2 NH3 (g)

L’enthalpie standard de cette réaction, supposée constante dans l’intervalle de température considéré est ∆H° = -92,5 kJ a) Partant des proportions stœchiométriques d’azote et d’hydrogène, on obtient à l’équilibre, 36 % d’ammoniac (fraction molaire) dans le mélange gazeux, sous la pression P0 = 300 atm et à la température t0 = 450°C. Calculer KP à cette température b) En maintenant la température à 450°C, à quelle pression totale P’ faut-il porter le mélange gazeux pour obtenir, à l’équilibre 50 % d’ammoniac. c) En maintenant la pression à sa valeur initiale (P0 = 300 atm), à quelle température T’ doit-on porter le mélange pour obtenir à nouveau 50 % d’ammoniac à l’équilibre ?

Exo 6: La décomposition du sulfate de calcium CaSO4 donne lieu aux équilibres suivants : (1) CaSO4 (s) (2) SO3 (g)

CaO (s) + SO3 (g) SO3 (g) + ½ O2 (g)

KP1 (1400 K) = 8,3 x 10-6 atm; KP2 (1400 K) = 16,3 atm1/2.

a) Calculer la variance des équilibres (1) et (2) séparément, puis, réalisés ensemble. b) On chauffe à 1400 K un récipient dans lequel on a fait le vide et où a été introduit du sulfate de calcium. Quelles seront les valeurs des pressions partielles des différents constituants du mélange gazeux à l’équilibre ?

46

C-2) Série de TD n° 3 (2002-2003) Exo 1: On considère l’équilibre gazeux homogène suivant, à 500 K : 1 2

PCl5

PCl3 + Cl2

La constante KC de cet équilibre est égale 0,04 mol.L-1. Dans un récipient de 5 litres, on introduit à 500 K, 0,2 mol de PCl5, 0,4 mol de PCl3 et 0,3 mol de Cl2, 1°) Dans quel sens évolue la réaction ? 2°) Déterminer la composition du mélange à l’équilibre. 3°) Calculer la pression totale, ainsi que les pressions partielles à l’équilibre. 4°) Calculer les valeurs de KN, KP et KX à 500 K.

Exo 2: L’oxychlorure de carbone se dissocie en donnant lieu à l’équilibre suivant : COCl2 (g)

1 2

CO (g) + Cl2 (g)

+

∆H°

On chauffe du COCl2 pur à la pression de 1 atm, et on détermine la densité du mélange gazeux obtenu par rapport à l’air. On trouve : t1 = 400 °C ⇒ d1 = 2,82 t2 = 500 °C ⇒ d2 = 2,52 1°) Déterminer, pour chaque température, le coefficient de dissociation. 2°) Déduire du sens de variation de α avec la température le signe de ∆H°. 3°) Calculer la valeur numérique de KP à 500 °C. 4°) A cette température de 500 °C, on porte la pression à 2 atm. Quelle est la nouvelle valeur du coefficient de dissociation ? Interpréter le résultat obtenu.

Exo 3: A 820°C, on réalise l’équilibre en phase gazeuse suivant, la pression totale étant égale à 1 atm : COCl2 (g)

1 2

CO (g) + Cl2 (g)

+

∆H°

A cette température, KP est égale à 1 atm. 1°) Quelle est la composition du mélange à l’équilibre si on part de 5 moles de CO2 + 4 moles de H2 + 1 mole de H2O ? 2°) Quelle serait la composition du mélange si la pression totale était de 20 atm ? 3°) A 700°C, la valeur de KP est égale à 0,59 atm. Quel est le signe de ∆H° ? 4°) Déterminer la composition du mélange dans ce nouvel état d’équilibre, si les conditions initiales sont les mêmes qu’au 1°).

47 5°) Calculer l’enthalpie de réaction ∆H°, supposée constante entre 700 et 820°C.

Exo 4: 1°) Pour la réaction de décomposition thermique du carbonate de calcium :

CaCO3 (s)

CaO (s) + CO2 (g)

log Kp = 7,3 -

8500

(P en atmosphère).

T

a) Donner les relations entre KP et KC et entre KP et KX. b) A T constant, quel est l’effet d’une augmentation de pression sur KP, KC et KX ? c) Calculer la valeur numérique de ∆H. d) Quel est l’effet d’une augmentation de température sur l’équilibre ? 2°) En présence de carbone solide, le monoxyde de carbone peut se former à partir de l’équilibre suivant :

CO2 (g) + C (s)

2 CO (g)

log KP2 = 9,12 -

8922

(P en atmosphère) T On réalise la réaction de dissociation de CaCO3 en excès dans une enceinte fermée, en présence de carbone solide. Etablir la relation entre la température T et la pression partielle de CO à l’équilibre.

Exo 5: On étudie la dissociation du NO2 en phase homogène 2 NO2 (g)

2 NO (g) + O2 (g)

La pression dans tout l’exercice sera de 1 atmosphère. 1°) A t1 = 377°C, la dissociation de NO2 conduit à un mélange gazeux dont la densité par rapport à l’air est égale à 4/5 de celle de NO2. Les gaz étant parfaits, on demande : a) le coefficient de dissociation de NO2 b) la constante d’équilibre KP à 377°C 2°) A t2 = 490°C, la constante KP est égale à 32/7 atm. Calculer l’enthalpie de formation d’une mole de NO2 à partir de NO et de O2, l’enthalpie étant supposée constante dans cet intervalle de température.

Exo 6: 1°) On considère à t = 630°C, l’équilibre gazeux homogène

SO3 (g)

SO2 (g) + 1/2 O2 (g)

A l’équilibre, la pression dans le récipient est 2,5 atm et la masse volumique du mélange gazeux est ρ = 2,35 kg m-3.

48 a) Donner la composition en pourcentage molaire du mélange. b) En déduire KP. 2°) Toujours à 630°C, on fait réagir un mélange équimolaire de dioxyde de soufre et d’air; le taux de conversion de SO2 en SO3 est de 30 %. a) Calculer la valeur numérique de la constante d’équilibre :

SO2 (g) + 1/2 O2 (g) SO3 (g) b) Calculer la pression totale ainsi que les pressions partielles de O2. Exo 7: Calculer la variance des équilibres suivants : a) 2 HI (g)

H2 (g) + I2 (g) (on part de HI qu’on chauffe)

b) H2 (g) + I2 (g) c) H2O (l)

2 HI (g)

H2O (g)

d) CaO (s) + CO2 (g) e) 3 MnSO4 (s) f) SO3 (g)

g)

CaCO3 (s)

Mn3O4 (s) + 2 SO3 (g) + SO2 (g) (on part de MnSO4 qu’on chauffe)

SO2 (g) + ½ O2 (g)

3 MnSO4 (s) SO3 (g)

Mn3O4 (s) + 2 SO3 (g) + SO2 (g) SO2 (g) + 1/2 O2 (g)

D) Examens D-1) Examen Partiel du 1er semestre 2001-2002 Exo 1: On mélange 400 g d’une solution de H2SO4 à 98 % (densité 1,84) et 600 g d’une solution de H2SO4 60 % (densité 1,73). Calculer les concentrations massique, molaire, équivalente et molale de la solution finale.

Exo 2: On considère la formation du chloroforme (CHCl3 (l)) par chloration du méthane (CH4 (g)) suivant la réaction ci-dessous : CH4 (g) + 3/2 Cl2 (g)

CHCl3 (l) + 3/2 H2 (g)

(I)

1°) Calculer ∆H°298 de cette réaction : a) à partir des enthalpies standard de formation des composés intervenant dans la réaction b) en proposant une combinaison linéaire des équations suivantes : 1. C (gr) + 2 H2 (g)

CH4 (g)

2. CHCl3 (l) + 5/4 O2 (g)

CO2 (g) + ½ H2O (l) + 3/2 Cl2 (g)

∆H1 = -74,8 kJ ∆H2 = -373,2 kJ

49 3. H2 (g) + ½ O2 (g) 4. C (gr) + O2 (g)

∆H3 = -285,6 kJ

H2O (l)

∆H4 = -393,1 kJ

CO2 (g)

2°) En déduire ∆U°298, puis calculer ∆H° à 323 K (CHCl3 est liquide à cette température). 3°) On introduit 9,6 g de méthane dans un réacteur en présence de 4 moles de chlore. La réaction étant totale, calculer: a) la quantité de chaleur libérée à pression constante par la réaction (I) à 298 K ; b) la température atteinte par les composés présents dans le réacteur après la réaction. Le chloroforme se vaporise à 298 K, avec une enthalpie de vaporisation égale à + 31,4 kJ.mol-1.

Données: ∆H°f (CH4 (g)) = -74,8 kJ.mol-1 ; ∆H°f (CH4 (g)) = -74,8 kJ.mol-1 ; CP (H2 (g)) = 28,8 J mol-1 K-1 ;

CP (CH4 (g)) = 35,3 J mol-1 K-1 ;

CP (Cl2 (g)) = 33,9 J mol-1 K-1 ;

CP (CHCl3 (l)) = 114,2 J mol-1 K-1 ;

CP (CHCl3 (g)) = 65,7 J mol-1 K-1.

D-2) Examen Partiel du 1er semestre 2002-2003 Exo 1: On dispose de deux solutions d’acide nitrique (HNO3) : Solution A : % = 54 %, d = 1,33 ; Solution B : η = 2,584 molale, d = 1,08. 1°) Calculer les concentrations équivalentes des solutions A et B. 2°) Calculer la concentration molale de la solution A. 3°) Combien de millilitres de la solution A et combien de grammes de la solution B fautil mélanger pour obtenir 0,2 L d’une solution C de HNO3 20% ? 4°) Calculer la masse volumique de la solution finale (C).

Données: H = 1 ; N = 14 ; O = 16. Exo 2: Soit la réaction :

CH3OH (g) + H2 (g)

CH4 (g) + H2O (g)

1°) Calculer l’enthalpie standard de cette réaction: a) à partir des énergies de liaisons. On supposera que les énergies des liaisons O-H dans H2O et CH3OH sont égales, de même que celles des liaisons C-H dans CH4 et CH3OH. b) à partir des enthalpies standard de formation ; 2°) L’énergie libérée par la réaction d’hydrogénation de 2 moles de méthanol (calculée à partir des enthalpies standard de formation) est entièrement utilisée pour chauffer 30 g de H2O (état de glace, initialement à –10 °C), que l’on retrouve finalement à l’état de vapeur.

50 Calculer la température finale atteinte par H2O.

Données: °

°

°

∆H f [CH3OH (l)] = -238,6 kJ mol-1; ∆H f [H2O (l)] = -284,3 kJ mol-1; ∆H f [CH4 (g)] = -74,9 kJ mol-1; °

°

∆H vap [H2O (l)] = +72 kJ mol-1 ; ∆H vap [CH3OH (l)] = +37,4 kJ mol-1; EH−H = -436 kJ mol-1 ; EC−H = -414,3 kJ mol-1 ; EO−H = -450 kJ mol-1 ; EC−O = -343,2 kJ mol-1 ; CP (glace)

= 2,09 J K-1 g-1 ; CP (H2O (l)) = 4,18 J K-1 g-1 ; CP (H2O (g)) = 1,87 J K-1 g-1 ;

Chaleur latente de fusion de la glace (à 0 °C) : LF = 334,4 J g-1 ; Chaleur latente de vaporisation de l’eau à (100 °C) : LV = 2261,1 J g-1.

D-3) Examen de Chimie Physique Section PC1 (session de juillet 2002) Exo 1: On dissout 220 mg de CO2 pur dans 500 mL d’eau (on néglige les variations de volume). 1°) Calculer le pH de cette solution de H2CO3 (solution A). 2°) On ajoute 250 mL de solution aqueuse de NaOH 0,01 M. Calculer le pH de la solution obtenue (B). 3°) On dissout dans la solution B 100 mg de NaOH solide (sans changement de volume). Quel est le pH de la solution C ainsi préparée ? 4°) A la solution C, on ajoute 250 mL de solution aqueuse de NaOH 0,01 M. Déterminer le pH de la solution finale.

Données : H2CO3 : pKA1 = 6,4 ; pKA2 = 10,2 H : 1, C : 12, Na : 23, O : 16.

Exo 2: On considère la réaction de synthèse de l’ammoniac : N2 (g) + 3 H2 (g)

2 NH3 (g)

1-a) Déterminer à 25 °C, les chaleurs de réaction à pression constante (QP) et à volume constant (QV). 1-b) Calculer l’énergie de la liaison N-H dans NH3 ; 2-a) Etablir la loi de variation de l’enthalpie de réaction avec la température ; 2-b) En déduire les nouvelles valeurs de QP et de QV à une température de 500°C ; 3°) Partant des proportions stœchiométriques de N2 et H2, calculer la constante d’équilibre à 500°C et sous une pression de 600 atm, sachant que la proportion de l’ammoniac dans le mélange gazeux final est de 40 % molaire ; 4-a) Calculer la valeur de KP à 600 °C ;

51 4-b) En déduire la valeur de la pression totale (à 600 °C) si la fraction molaire de l’ammoniac est égale à 1/10 ; 4-c) Quelle serait la valeur de la fraction molaire de l’ammoniac, si la pression totale était maintenue à 600 atm ? 5°) Discuter ces variations de la fraction molaire de NH3 avec la pression et avec la température.

Données: ∆H°f (NH3) (g) = - 47,65 kJ mol-1 ; CP (H2) (g) = 29,9 J mol-1 K-1 ;

CP (N2) (g) = 30,3 J mol-1 K-1 ;

CP (NH3) (g) = 52,2 J mol-1 K-1 ;

N2 (g)

2 N (g)

∆H1 = 940,5 kJ

H2 (g)

2 H (g)

∆H1 = 435,5 kJ.

R = 8,31 J mol-1 K-1.

D-4) Examen de Chimie Physique Section PC1 (session d’octobre 2002) Exo 1 Soient les équilibres : (1)

3 MnSO4 (s)

(2)

2 SO3 (g)

Mn3O4 (s) + 2 SO3 (g) + SO2 (g) 2 SO2 (g) + O2 (g)

Dans un récipient vide d’air, on a placé un excès de MnSO4 et porté la température à 1300 K. A l’équilibre la pression totale est de 78,3 cm de Hg et la pression partielle de SO2 est de 55,5 cm de Hg. 1-1) Déterminer le coefficient de dissociation α de SO3 ; 1-2) Calculer les pressions partielles de SO3 et de O2 ; 1-3) En déduire les constantes d’équilibres KP1 et KP2 ; 1-4) Calculer la densité par rapport à l’air du mélange gazeux final ; 1-5) Quelle est la variance de ce système ?

Données : O : 16 ; S : 32 Exo 2: Le produit de solubilité de l’iodure de plomb (PbI2) est de 10-8 M3. 2-1) Quelles sont les concentrations des ions Pb2+ et I- dans une solution aqueuse saturée en PbI2 ? 2-2) Calculer la valeur de la solubilité de PbI2 dans une solution aqueuse contenant : a) 0,1 M de KI b) 0,1 M de Pb(NO3)2 2-3) On plonge un fil de plomb dans :

52 a) une solution aqueuse saturée en PbI2 ; b) une solution aqueuse de KI 0,1 M, saturée en PbI2 ; c) une solution aqueuse de Pb(NO3)2 0,1 M, saturée en PbI2 . Calculer, dans chaque cas, le potentiel pris à 25°C par le couple Pb2+/Pb.

Donnée : E° (Pb2+/Pb) = −0,126 V/ENH. D-5) Examen de Chimie Physique Section PC1 (session de juillet 2003) Exo 1: Les produits de solubilité du bromure d’argent (AgBr) et de l’isothiocyanate d’argent (AgSCN) à 25°C, sont respectivement 7,7 x 10-13 et 1,16 x 10-12. On négligera l’action des ions Br- et SCN- sur l’eau. 1-1) Calculer la solubilité de AgBr, ensuite celle de AgSCN dans l’eau pure. 1-2) Calculer la solubilité de AgBr et celle de AgSCN ensembles dans l’eau pure. Comparer les valeurs obtenues avec les solubilités des sels pris individuellement et conclure.

Exo 2: On réalise la combustion du dioxyde de soufre (SO2) en trioxyde de soufre (SO3) dans un réacteur de volume constant :

SO2 (g) + 1/2 O2 (g) SO3 (g) 2-1) On part d’un mélange stœchiométrique de SO2 et d’oxygène, initialement à la pression atmosphérique et à 630 °C. Le taux de conversion de SO2 en SO3 est égal à 0,6. 2-1-1) Calculer la pression totale. 2-1-2) Evaluer la constante d’équilibre. 2-2) On part d’un mélange de SO2 et d’air, dans lequel SO2 et O2 sont dans les proportions stœchiométriques. La pression initiale est toujours égale à 1 atm et la température à 630 °C. La pression totale s’établit à 0,925 atm. 2-2-1) Calculer le taux de conversion de SO2 en SO3. 2-2-2) Comparer ce nouveau taux de conversion avec celui obtenu au 2-1 et expliquer. 2-2-3) Quelle est la densité du mélange gazeux par rapport à l’air ? 2-2-4) Calculer la variance de l’équilibre.

Données: Masses atomiques :

S : 32 ;

O : 16 ;

N : 14.

On prendra la composition volumétrique suivante pour l’air : 20 % O2 et 80 % N2.

D-6) Examen de Chimie Physique Section PC1 (session d’Octobre 2003) Exo 1:

53 On dispose des solutions suivantes : -

NH4OH 0,1 M (solution A);

-

Ca(OH)2 (dibase forte) 0,005 M (solution B);*

-

HCl 0,1 M (solution C).

-

HCOOH 0,01 M (solution D).

1-1) Calculer le pH de chacune des solutions A, B, C et D. 1-2) Calculer le pH des solutions obtenues en mélangeant : 1-2-1) : 100 mL de A et 25 mL de C ; 1-2-2) : 10 mL de A et 100 mL de D.

On donne : NH4OH : constante de basicité = 1,79 x 10-5 ; HCOOH : constante d’acidité = 1,77 x 10-4.

Exo 2: On considère la pile représentée par la chaîne suivante : Al

Al2(SO4)3 0,1 M

CuSO4 0,1 M Cu

2-1) Faire le schéma de la pile, en indiquant les polarités, le sens de circulation du courant, l’anode, la cathode, etc. 2-2) Ecrire la réaction globale qui a lieu quand la pile débite. 2-3) Calculer la constante d’équilibre de la réaction globale. 2-4) Calculer les concentrations finales des ions lorsque la pile est usée.

On donne: E°Cu2+/Cu = 0,34 V/ENH;

E°Al3+/Al = -1,66 V/ENH.

D-7) Examen Partiel du 1er semestre 2003-2004 Exo 1 : On plonge un glaçon de 36 g à – 10 °C dans 90 g d’eau à 50 °C. La glace fond. 1-1)

En supposant le système adiabatique, calculer la température finale de l’eau (θF).

1-2)

Quelle est la quantité de chaleur nécessaire pour amener la masse d’eau finale de θF à 150 °C (état vapeur) ?

Données : CP (H2O (l)) = 76 J mol-1 K-1 ; CP (glace) = 38 J mol-1 K-1 ; CP (H2O (vap)) = 34 J mol-1 K-1 ; Chaleur latente de fusion de la glace : LF = 6 kJ mol-1 ; Chaleur latente de vaporisation de l’eau : LV = 40,7 kJ mol-1. *

Dans l’épreuve, une valeur de 0,1 M avait été donnée pour la concentration de Ca(OH)2. Nous avons modifié ici la concentration pour proposer une valeur inférieure à la solubilité de Ca(OH)2 dans l’eau à 20°C.

54

Exo 2 : Pour la réaction en phase gazeuse, à 250 °C :

PCl5

PCl3 + Cl2

KP = 1,7 atm

On introduit 0,5 mol de pentachlorure de phosphore (PCl5) et 0,5 mol de chlore dans un récipient de volume 2 litres. La température est maintenue à 250°C. Calculer, à l’équilibre : -

la fraction molaire de chaque constituant ;

-

la pression totale ;

-

la densité du mélange gazeux par rapport au chlore.

On donne :

P : 31 ; Cl : 35,5.

D-8) Examen de Chimie Physique Section PC1 (session de juillet 2004) Exo 1 : La combustion de 78 g de benzène liquide à 25 °C à pression constante (1 atm) dégage 3265 kJ. 1°) Ecrire l’équation de la réaction de combustion. 2°) Calculer la variation d’énergie interne de la réaction de combustion à 25°C . 3°) Calculer l’enthalpie de la réaction à 80 °C. - le benzène étant liquide ; - le benzène étant gazeux. 4°) Calculer la quantité de chaleur dégagée par la combustion à volume constant de 3,9 g de C6H6 (g) à 80 °C.

Données : Cp (C6H6 (l)) = 148 J mol-1 K-1; CP (O2 (g)) = 29 J mol-1 K-1;

Cp (H2O (l)) = 76 J mol-1 K-1;

CP (CO2 (g)) = 29 J mol-1 K-1;

∆HVAP 80 °C (benzène) = 30,84 kJ.mol-1

Exo 2 : On dissout 2,76 g de NaNO2 dans un litre d’eau, sans variation de volume. 1°) Calculer le pH de la solution A ainsi obtenue. 2°) On ajoute à 100 mL de A : -

10 mL de HCl 0,1 M (solution B);

-

20 mL de HCl 0,1 M (solution C);

-

30 mL de HCl 0,1 M (solution D);

-

40 mL de HCl 0,1 M (solution E) ;

-

50 mL de HCl 0,1 M (solution F).

Calculer le pH de chacune des solutions B, C, D, E et F.

55 pKA (HNO2/NO2-) = 3,4

Données : Na : 23 ;

N : 14 ;

O : 16.

D-9) Examen de Chimie Physique - PC1 - Session de juillet 2005 Exo 1 : On considère la réaction de synthèse du méthanol, à partir d’un mélange équimolaire de CO et H2 (gaz à l'eau) :

CO (g) + 2 H2 (g)

CH3OH (g)

∆H° = -90960 J

1°) Calculer l’enthalpie standard de combustion du méthanol, à partir de celles de H2 et CO. 2°) A 25 °C, la constante d’équilibre (KP1) est 2,51 x 104 atm-2. Calculer alors la pression totale à appliquer au mélange pour obtenir un degré de dissociation de 99% de l’hydrogène.. 3°) On porte la température à 200°C. Calculer la nouvelle constante d’équilibre (KP2). 4°) Si la pression totale ne varie pas, quel est le taux de conversion correspondant ? 5°) Calculer la pression totale à appliquer au mélange pour retrouver le coefficient de dissociation de 99% de H2. 6°) Discuter ces variations du taux de conversion avec la pression et avec la température.

Données : ∆H°combust° (CO (g)) = -282,7 kJ.mol-1 ; ∆H°combust° (H2 (g)) = -285,7 kJ.mol-1 ; R = 8,314 J.mol-1K-1.

Exo 2 : On dispose des solutions aqueuses suivantes : Solution A : acide formique (HCOOH), C1 = 0,4 M et KA1 = 1,8 x 10-4 ; Solution B : acide acétique (CH3COOH), C2 = 0,4 M et KA2 = 1,8 x 10-5. 1°) Calculer le pH de chacune des solutions A et B. 2°) On mélange 100 mL de A et 100 mL de B. Calculer le pH de la solution D ainsi préparée. 3°) On ajoute à la solution D 3,2 g de NaOH, en négligeant les variations de volume. Calculer le pH de la solution E ainsi obtenue. 4°) Quel serait le pH si on avait ajouté seulement 1,6 g de NaOH à la solution D ? Quelles seraient les concentrations molaires de CH3COONa et HCOONa formés ?

Données : Masses atomiques: Na : 23 ; O : 16 ; H : 1.

56

Corrections des TD, partiels et examens proposés durant les années 2001-2002 et 2002-2003 A-1) Série de TD n° 1 - 2001-2002 Exo 1 : Solution A : V = 15 cm3 ; m (Na2CO3) = 1,65 g ;

Fraction massique de Na2CO3:

ρ = 1170 kg m-3 (1170 g L-1)

x = m / (m + me) = m/ms = m / (ρV)

a) Concentration massique de la solution A:

AN : x = 1,65 / (1170 x 0,015) = 0,094.

Cm = m / V

d) Pourcentage massique de Na2CO3 :

A.N: Cm = 1,65 / 0,015 = 110 g L-1.

% = (m/ms) x 100 = 100 x

b) Concentration molaire de A:

A.N: % = 165 / (1170 x 0,015) = 9,4 %.

CM = n / V = Cm /M

e) Concentration équivalente :

A.N: CM = 110 / 106 = 1,038 M.

Céq = néq-g / V = pCM

c) Fraction molaire de Na2CO3:

Na2CO3 est une dibase:

X = n / (n + ne) = (m / M) / [m / M + (ρV – m) / Me]

Na2CO3 + 2 H+

A.N:

2Na+ + CO2 + H2O: p = 2

A.N: Céq = 2 x (110/106) = 2,075 éq-g/L.

X = 1,65/106/[1,65/106 + (1170 x 0,015 – 1,65)/18] X = 0,017.

Exo 2 : HNO3 : % = 78,07; d = 1,454

c) Concentration molaire :

a) Masse de soluté :

CM = n/V = m/M/V

% = 100 x m/ms ⇔ m = (%/100) x ms

1ère méthode : sachant qu’un litre de solution

% = (%/100) x ρV = (%/100) x ρe x d x V

contient 1135,14 g de soluté, on a :

A.N: m = (78,07/100) x 1000 x 1,454 x 1

Pour V = 1 L, m = 1135,14 ;

m = 1135,14 g

A.N: CM = 1135,14/63/1 = 18,018 M.

b) Molalité :

2ème méthode : CM = m/M/V; or m = (%/100) x ms:

η = n / me(kg) η = m / M / (ms - m) = 1000 x / M / (1 - x), avec x = %/100 et M = 63 g mol-1. A.N : η = 1000 x 0,7807 / 63 / (1 - 0,7807)

⇒ CM = (%/100) x (ms/V)/M = (%/100) x ρ / M

η = 56,5 molale.

CM = (%/100) x ρe x d / M A.N: CM=(78,07/100) x 1,454 x 1000/63 = 18,018 M.

d) Volume d’eau à ajouter (Ve) : Solution A: VA = 55,52 mL ; CA = 18,018 M

57 Solution B : VB = VA + Ve; CB = 0,5 M (car

A.N: Ve = 55,52 x (18,018/0,5 - 1) = 1945,21 mL

HNO3 est un monoacide et donc p = 1).

e) Densité de la solution B :

La dilution par de l’eau ne modifiant pas le

d = ρ/ρe = msB/VB/ρe = (msA + me)/V/ρe

nombre de moles de soluté, on peut écrire :

d = (msA + me)/VB/ρe = (ρAVA + ρeVe)/VB/ρe

CAVA = CBVB = CB x (VA +Ve)

d = [(ρA /ρe) x VA + Ve]/(VA+Ve)

⇒ Ve = VA x (CA/CB – 1)

A.N: d = [1,454 x 55,52 + 1945,21] / (55,52 + 1945,21) d = 1,013.

Exo 3 : Pentane : ρ1 = 0,6263 g mL-1 ; V1 = 2V Hexane: ρ2 = 0,6638 g mL-1 ; V2 = 5V

ρ = ms/V = ms/(V1 + V2) = (ms1 + ms2)/(V1 + V2) = (ρ1V1 + ρ2)/(V1 + V2) ρ= (ρ1 x 2V + ρ2 x 5V)/(2V + 5V) = (2ρ1 + 5ρ2)/7 A.N: ρ = (2 x 0,6263 + 5 x 0,6638)/7 = 0,6531 g mL-1

Exo 4 : Solution 1 : η 1 = 1,001 molale ; ms1 = ?

Solution 2 : η 2 = 6,006 molale ; ms2

Solution finale : η = 3,861 molale ; ms = 240 g ; ρ = 1,2816 g mL-1. En mélangeant les deux solutions de CaCl2, on peut écrire que la masse de solution finale est la somme des masses des solutions 1 et 2, et que la masse de soluté dans la solution finale est la somme des masses de soluté contenu dans les solutions 1 et 2 : (1) ms1 + ms2 = ms (2) x1 ms1 + x2 ms2 = x ms (x est la fraction massique de CaCl2) La molalité et la fraction massique sont reliées par la relation :

η = 1000 x /M/(1 − x) ⇔ x = η x M/(1000 + η x M) Solution 1 : x 1 = 1,001 x 111 / (1000 + 1,001 x 111) = 0,1 Solution 2 : x 2 = 6,006 x 111 / (1000 + 6,006 x 111) = 0,4 Solution finale : x = 3,861 x 111 / (1000 + 3,861 x 111) = 0,3 En mettant les valeurs numériques de x dans l’équation (2), on obtient le système : (1)

ms1 + ms2 = ms = 240 g

(2)

0,1 ms1 + 0,4 ms2 = 0,3 ms = 72 g

58 La résolution de ce système donne : ms1 = 80 g et ms2 = 160 g. CaCl2 est un sel (Ca2+, 2Cl-) p = 2. Par conséquent Céq = 2 x CM. Le calcul précédent a montré que la masse de soluté dans la solution finale est : m = 0,3 ms = 72 g Céq = 2 x CM = 2 x m/M/V, avec M = masse molaire de CaCl2. Céq = 2 x m / M / (ms / ρ) A.N: Céq = 2 x 72 x 1281,6 /240/111 = 6,93 éq-g L-1.

Exo 5 : Solution B : KOH 20,16% ; ms = ? ;

Solution A : Acide

La base réagit totalement avec l’acide. A la neutralisation, le nombre d’équivalents-grammes de base doit être égal au nombre d’équivalents-grammes d’acide : NBVB = NAVA. KOH est une monobase : p = 1. NBVB = mKOH/MKOH = (%/100) x msKOH/MKOH. msKOH = (MKOH x 100 / 20,16) x (NBVB) = (MKOH x 100 / 20,16) x (NAVA) msKOH = NAVA x 56 / 0,2016. ms KOH = (56/0,2016) x η x ms/(1000 + η x MHCl)

a) H3PO4 : VA = 200 mL ; CA = 5,39 M. H3PO4

+

3H +

PO43-

⇒p=3

A.N: ms KOH = (56 / 0,2016) x 8,03 x 150 / (1000 +

ms KOH = NAVAx56/0,2016 = 3 CAVAx56/0,2016

8,03 x 36,5) = 258,75 g.

A.N: ms KOH = 3x5,39x0,2x56/0,2016 = 898,33 g

d) HNO3 : VA = 75 mL, Cm = 0,126 g mL-1

-1

b) H2SO4 : VA = 125 mL ; NA = 3,92 éq-g L . H2SO4

+

2H +

SO42-

:⇒p=2

ms KOH = NAVA x 56 / 0,2016 A.N: ms KOH = 3,92x0,125x56/0,2016 = 136,11 g

c) 150 g HCl 8,03 molale

η = 1000 x mHCl / MHCl / (ms - mHCl) ⇒ mHCl = η x MHCl x ms / (1000 + η x MHCl) HCl étant un monoacide, p = 1. NAVA = néq-g HCl = mHCl / MHCl NAVA = η x ms / (1000 + η x MHCl) ms KOH = NAVA x 56 / 0,2016

NAVA = CmVA / MHNO3

⇒ ms KOH = CMVA / MHNO3 x 56 / 0,2016 A.N: ms KOH = (0,126 x 75 / 63) x 56 / 0,2016 ms = 41,67 g.

e) HCOOH : ms = 239 g, X = 0,04. X = n / (n + ne) = (m / M) / [m / M + (ms − m) / Me] X = Me x m / [Me x m + M x (ms - m)]

⇒ m = 46 ms X / (18 + 28 X) HCOOH

H+ + HCOO- ⇒ p = 1.

NAVA = m/M = ms X/(18 + 28 X) ms KOH = NAVA x 56/0,2016 ⇒

59 ms KOH = (56/0,2016) x ms X/(18 + 28 X)

NAVA = nég.g KHCO3 = (% / 100) x ρV / MKHCO3

A.N:

ms KOH = NAVA x 56 / 0,2016 ⇒

ms KOH = (56 / 0,2016) x 239 x 0,04/(18 + 28 x 0,04)

ms KOH = 56 / 0,2016 x (% / 100) x ρV / MKHCO3

= 138,89 g.

A.N: ms KOH = (56/0,2016) x 0,1 x 1,0674 x 50/100 -1

f) KHCO3 10 %: V = 50 mL ; ρ = 1,0674 g mL . HCO3

-

+

H + CO3

2-

ms KOH = 14,825 g.

⇒ p=1

Exo 6 : 6. A :

1 litre de solution d’eau de javel contient 0,1

H2O2 : V1 = 20 cm3 ; N1 = ?

mole de NaClO et dégage donc 0,1 mole de Cl2,

KMnO4 : V2 = 40 cm3 ; C2 = 31,6 g L-1

suivant la réaction :

A la neutralisation, N1V1 = N2V2

NaClO + 2 HCl

⇒ N H2O2 = N1 = N2V2/V1.

NCl2 = 0,1 mole ⇒ VCl2 = 0,1 x 22,4 = 2,24 L.

+

KMnO4 + 8H + 5e

-

+

2+

K + Mn + 4H2O (p = 5)

Cl2 + 2 NaCl + H2O

T = 2,24 degrés chlorométriques.

NKMnO4 = 5 CM = 5 Cm/M (M = masse molaire

Céq = p x m NaClO / (MV) = 2 x 1,49 / 74,5 / 0,2

de KMnO4)

Céq = 0,2 éq-g L-1.

NH2O2 = N1 = 5 CmV2 / V1 / M

CM = Céq / p = 0,2 / 2 = 0,1 M.

H2O2

2 H+ + O2 + 2e- ⇒ p = 2

Solution 1: T1 = 2,24; V1 = 200 mL ;

CH2O2 = NH2O2 / 2 = 5 CmV2 / (2 M V1)

Solution 2: T2 = 1,18; V2 = V1 + Ve.

CH2O2 = 5 x 31,6 x 40 / (2 x 158 x 20) = 1 mol L-1.

1 litre de solution dégage T litres de Cl2

1 litre de cette solution d’eau oxygénée contient

⇒ nCl2 = T / 22,4

donc 1 mole de H2O2 et dégage 0,5 mole de O2,

On sait qu’une mole de NaClO dégage 1 mole

d’après la réaction :

de Cl2 (d’après réaction).

H2O2

n NaClO = nCl2 = T / 22,4 ⇒ CM = T / 22,4.

H2O + ½ O2

nO2 = 0,5 mole

Le principe de la conservation du nombre de

⇒ VO2 (dans les CNTP) = 0,5 x 22,4 = 11,2 litres

moles permet d’écrire :

T = 11,2 volumes.

C1V1 = C2V2 ⇒ T1V1 / 22,4 = T2V2 / 22,4

6. B :

⇒ T1V1 = T2V2

NaClO : V = 200 mL ; m = 1,49 g.

T1V1 = T2 x (V1 + Ve) ⇒ Ve = T1V1 / T2 - V1

Si 200 mL de solution contiennent 1,49 g de

= V1 x (T1 / T2 - 1)

soluté, 1 L de solution contient :

A.N: Ve = 200 x (2,24 / 1,18 - 1) = 179,7 mL.

n = 1,49x1000/200 /74,5 = 0,1 mol

60

A-2) Série n°1 - 2002-2003 On appellera m la masse de soluté, me la masse de solvant (eau), ms la masse de solution, V le volume de solution, ρ la masse volumique de la solution, n le nombre de moles de soluté, ne le nombre de moles d’eau et M la masse molaire du soluté.

Exo 1 : a) Concentration massique (Cm) :

Xe = ne / (n + ne) = me x M / (me x M + m x Me)

Cm = m/V ;

A.N: X = 20,5 x 18/(20,5 x 18 + 500 x 98);

ρ = ms/V = (m + me)/V ⇒ V = (m + me) / ρ

X = 0,0075.

Cm = m x ρ /(m + me).

Xe = 500 x 98 / (500 x 98 + 20,5 x 18) ;

A.N: Cm = 20,5 x 1025 /(20,5 + 500)

Xe = 0,9925.

Cm = 40,37 g L-1.



Fraction massique (x) :

x = m/(m + me) = m / (m + me) ;

b) Concentration molaire (CM) :

xe = me/(m + me) = me / (me + m) ;

CM = Cm/M ; -1

A.N: CM = 40,37/98 = 0,412 mol L .

A.N: x = 20,5/(20,5 + 500) = 0,0394.

xe = 20,5/(20,5 + 500) = 0,9606.

c) Concentration équivalente (Céq) : Céq = p x CM , avec p = nombre de protons mis

e) Concentration molale ( η ) :

en jeu.

η = n/me(kg) = m / M / me(kg) ;

H2SO4 → 2H + +

SO42- ;

⇒ p=2

A.N: η = 20,5/98/0,5 = 0,418 molale. -1

A.N: Céq = 2 x 0,412 = 0,824 éq-g L .

d) Fraction molaire (X) :

f) Densité: d = ρ/ρe : A.N : d = 1,025/1 = 1,025.

X = n/(n + ne) = m x Me/(m x Me + me x M) ;

Exo 2 : Solution de H3PO4 : M = 98 g mol-1 ; d = 1,71 ; % = 85%.

 Concentration équivalente : (Céq) : Céq = p x CM = p x (m / M) / V

⇒ Céq = p x ms x (% / 100) / M / V % = 100 m / ms ⇒ m = ms % / 100 H3PO4 → 3H+ + PO43- ;

d = ρ / ρe ⇒ ρ = ms / V = d x ρe

p = 3;

A.N: Céq = 3 x 1,71 x 1000 x 85/(100 x 98) = 44,49 éq-g L-1.

⇒ Céq = p x d x ρe x % / 100 / M

61



Concentration molaire : (CM) :

CM = Céq/p = d x ρe x %/100 / M; A.N: CM = 1,71 x 1000 x 85 / (100 x 98) = 14,83 mol L-1.

 Préparation de 100 litres d’une solution 2,97 M : Ci = 14,83 M

Cf = 2,97 M

⇒ (dilution) Vi = ?

Vf = Vi + Ve = 100 L.

La dilution n’entraînant pas une modification du nombre de moles de soluté, on peut écrire : CiVi = CfVf ⇒ Vi = CfVf/Ci , et Ve = Vf – Vi = Vf (1 - Cf/Ci) A.N: Vi = 2,97 x 100 /14,83 # 20 L ; Ve # 80 L.

 Pourcentage en masse de la nouvelle solution : % = (m/ms) x 100 = 100 x CfVf x M / (di x ρe x Vi + ρe x Ve) A.N: % = 100 x 2,97 x 100 x 98/(1,71 x 1000 x 20 + 1000 x 80) = 25,5%.

Exo 3 : Solutions de Na2CO3 (M = 106 g mol-1) % = 40% Solution A d = 1,824 VA = 220 mL

Céq = 16,17 éq-g L-1 Solution B d = 1,775 Solution C msB = 522 g

η = 4 molale d = 1,173 VC = 515,6 mL

a) Concentration molaire des solutions A, B et C :



CA = n/V = (m / M) / V = (% / 100) x mS / V / M = (%/100) x ρe x d / M

A.N: CA = (40 / 100) x 1824 / 106 = 6,883 M.



CB = Céq / p = Céq / 2

A.N: CB = 16,17/2 = 8,085 M.



η = (1000 x CC)/( ρe x d – M x CC) ⇒ CC = η x ρe x d / (1000 + M x η ) ;

A.N: CC = 4 x 1173 / (1000 + 106 x 4) = 3,295 M. b) Concentration molaire de la solution finale : Le nombre de moles de soluté dans la solution finale est égal à la somme des nombres de moles de soluté dans les solutions A, B et C. nA + nB + nC = nf ⇔ CAVA + CBVB + CCVC = CfVf ⇔ Cf = (CAVA + CBVB + CCVC) / (VA + VB + VC) VB = msB / ρB = msB / (ρe x dB) A.N: Cf = (6,883 x 220 + 8,085 x (522 / 1,775) + 3,295 x 515,6) / (220 + 522 / 1,775 + 515,6) = 5,43 M.

62

Exo 4 : On peut exprimer le nombre de mol et la masse de chaque constituant en fonction de la masse d’eau (en kg) :

η H2SO4 = nH2SO4/me(kg) ⇒ nH2SO4 = η H2SO4 x me ⇒ mH2SO4 = η H2SO4 x me x MH2SO4 η HNO3 = nHNO3/me(kg) ⇒ nHNO3 = η HNO3 x me ⇒ m HNO3 = η HNO3 x mex M HNO3 mHCl/mS = 0,1 ⇒ mHCl = 0,1 x (mHCl + mH2SO4 + mHNO3 + 1000 x me)

⇒ nHCl = ( η H2SO4 x MH2SO4 x me +

η HNO3 x M HNO3 x me + 1000 x me) / (9 x MHCl)

nombre total de moles solution: nT = (nHCl + nH2SO4 + nHNO3 + 1000 x me/18) nT = ( η H2SO4 x [1 + MH2SO4/(9 MHCl)] + η HNO3 x [1 + MHNO3/(9 MHCl)] + 1000 x [1/18 + 1/(9 MHCl)]) x me nT = 62,6882 x me (moles).

Xi = ni/nT A.N:

η H2SO4 / 62,6882 = 2,02 / 62,6882 = 0,032 XHNO3 = η HNO3 / 62,6882 = 1,23 / 62,6882 = 0,020 XHCl = [( η H2SO4 x MH2SO4 + η HNO3 x M HNO3 + 1000) / (9 x MHCl)]/62,6882 = 0,062 XH2SO4 =

XH2O = 1000 / 18 / 62,6882 = 0,886. Exo 5 : Solutions de H2SO4 (M = 98 g mol-1)

a) Masses de A et B à mélanger :

η B = 4 molale

%A= 90% Solution A

Solution B msA = ?

ρB = 1,205 g mL-1 VB = 1,5 L

CéqC= 6,4 éq-g L-1 Solution C VC = 2 L

La masse de soluté dans la solution C est égale à la somme des masses de soluté dans les solutions A et B : mA + mB = mC ;

(1)

mA = %A x msA /100;

(2)

η B = 1000 x mB / M/(ρB x V– mB) ⇒ mB = η B x ρB x V x M / (1000 + η B x M);

(3)

mC = CéqC x VC x M /p = CéqC x VC x M/2

(4)

(1) + (2) + (3) + (4)⇒ msA = (100 / %A) x [CéqC x VC x M /2 -

η B x ρB x V x M/(1000 + η B x M)]

63 A.N: msA = (100 / 90) x [6,4 x 2 x 98 / 2 - 4 x 1205 x 1,5 x 98/(1000 + 4 x 98)] = 131,32 g.

b) Masse de C : msC = msA + msB = msA + ρB x VB ;

d) Concentration massique de Na2CO3 :

A.N: msC = 131,32 + 1205 x 1,5 = 1938,82 g.

Solution C: Céq1 = 6,4 éq L-1;

c) Masse de NaOH :

NaOH

Solution C : Céq1 = 6,4 éq-g L-1; NaOH

V1 = 15 mL;

Céq2 = 3,2 éq-g L-1 ; V2 = ?

V1 = 15 mL;

Céq2 = ? ; V2 = 30 mL.

A la neutralisation, on a : Céq1 x V1 = Céq2 x V2

⇒ Céq2= Céq1 x V1 / V2 ;

A la neutralisation, le nombre d’équivalents-

Na2CO3 est une dibase (p = 2)

grammes de base est égal au nombre

⇒ Céq2= 2 x CM = 2 x Cm / M

d’équivalents-grammes d’acide :

⇒ Cm = Céq2 x M / 2 = (Céq1 x V1 / V2) x M / 2.

Céq1 x V1 = Céq2 x V2 ⇒ V2 = Céq1 x V1 / Céq2 ;

A.N: Cm = (6,4 x 15 / 30) x 106 / 2 = 169,6 g L-1.

A.N: V2 = 6,4 x 15 / 3,2 = 30 mL.

Exo 6: MnO4- + 8 H+ + 5 e5 Fe2+

Mn2+ + 4 H2O

5 Fe3+ + 5 e-

MnO4- + 8 H+ + 5 Fe2+

Mn2+ + 5 Fe3++ 4 H2O

a) Masse de KMnO4 (Céq = 1 éq-g L-1 ) : CM = Céq / p ; Cm = CM x M = (Céq /p) x M ; A.N: Cm = (1/5) x 158 = 31,6 g b) Masse de FeSO4, 7H2O : A la neutralisation, on a : Céq (KMnO4) x V(KMnO4) = Céq (FeSO4, 7H2O) x V(FeSO4, 7H2O) = nég.g (FeSO4, 7H2O) Puisque p = 1 dans le cas de (FeSO4, 7H2O), n(FeSO4, 7H2O) = Céq (KMnO4) x V(KMnO4). m(FeSO4, 7H2O) = Céq (KMnO4) x V(KMnO4) x M(FeSO4, 7H2O) ; A.N: m(FeSO4, 7H2O) = 1 x 0,5 x 278 = 139 g.

Exo 7-A: L’information signifie qu’un litre de la solution contenue dans le flacon peut dégager dans les CNTP 8 litres de chlore, suivant la réaction : NaClO + 2 HCl

Cl2 + NaCl + H2O

Si V solution = 1 L, VCl2 = TCl = 8 litres ⇒ nCl2 = Tv/Vm mole.

64 Vm = volume molaire des gaz dans les CNTP, et TCl = titre volumique de la solution de NaClO. D’après la stœchiométrie de la réaction, 1 mole de Cl2 est dégagée par 1 mole de NaClO ; Par conséquent, nH2O2 = nCl2 = TCl/Vm. Puisque le volume de la solution est égal à 1 litre, et que l’hypochlorite de sodium met en jeu deux électrons (p = 2), les concentrations molaire, équivalente et massique sont : CM = TCl / Vm ;

Céq = 2CM = 2 x TCl / Vm ;

A.N: CM = 8/22,4 = 0,357 M ; Solution initiale :

Cm = M x CM = 74,5 x TCl / Vm ; -1

Céq = 2CM = 0,714 éq-g L ;

Ci = 0,357 M Vi = ?

solution finale :

Cm = M x CM = 26,61 g L-1. Cf = 0,1 M Vf = 1 L

CiVi = CfVf ⇒ Vi = CfVf/Ci ; A.N: Vi = 0,1 x 1 / 0,357 = 0,28 litres.

Exo 7-B : Relations entre les différentes expressions de la concentration et le titre volumique d’une solution d’eau oxygénée : H2O2

H 2O + ½ O 2

Si V solution = 1 L, VO2 = TV = 8 litres ⇒ nO2 = Tv/Vm mol Vm = volume molaire des gaz dans les CNTP, et TV = titre volumique de la solution de H2O2. D’après la stœchiométrie de la réaction, 1 mole de O2 est dégagée par 2 moles de H2O2 ; Par conséquent, nH2O2 = 2 x nO2 = 2 x Tv/Vm. Puisque le volume de la solution est égal à 1 litre, et que l’eau oxygénée met en jeu deux électrons, les concentrations molaire, équivalente et massique sont : CM = 2 x Tv/Vm ;

Céq = 2CM = 4 x Tv/Vm ;

Cm = M x CM = 68 x Tv/Vm ;

Solution A : mA = 0,068 g ; VA = 250 mL ⇒ CA = Cm/M = mA / M / VA = 0,068/0,25/34 = 0,008 M ; Solution B : VB = ? Céq B = 0,1 N ⇒ CB = Céq B / 2 = 0,05 M ; Solution C : VC = VA + VB ; TV = 0,112 volume ⇒ CC = 2 x 0,112 / 22,4 = 0,01 M. Le nombre de moles de soluté dans la solution C est égal à la somme des nombres de moles de soluté dans les solutions A et B : CAVA + CBVB = CCVC = CC x (VA + VB) ⇒ VB = VA x (CC - CA) / (CB – CC). A.N: VB = 250 x (0,01 – 0,008) / (0,05 – 0,01) = 12,5 mL.

B-1) Série de TD n° 2 (2001-2002)

65

Exo 1: Equation d’état d’un gaz parfait : PV = nRT Plaçons-nous dans les CNTP : P = 1 atm (101325 Pa) ; V / n = 22,414 L mol-1 ; T = 273,15 K R = PV / nT A.N: R en L atm mol-1K-1 : R = 1 x 22,414 / 273,15 = 0,082 L atm mol-1K-1. R en J mol-1K-1 : R = 101325 x 22,414 x 10-3 / 273,15 = 8,314 J mol-1K-1 R en cal mol-1K-1 : R = 8,31447 / 4,185 = 1,987 cal mol-1K-1.

Exo 2: Le gaz à l’eau a la composition suivante (pourcentage en masse). H2

CO

N2

CO2

CH4

6,43

67,82

10,71

14,02

1,02

On peut prendre comme base de calcul 100 g de mélange, puis calculer la masse de chaque constituant, en déduire son nombre de moles, sa fraction molaire, etc. % volumique = % molaire = 100 x fraction molaire. H2

CO

N2

CO2

CH4

Σ

mi (g)

6,43

67,82

10,71

14,02

1,02

100

ni (mol)

3,2150

2,4221

0,3825

0,3186

0,0637

6,4019

Xi = ni /(Σni)

0,5022

0,3783

0,0597

0,0498

0,0100

1

Pourcentage

50,22

37,83

5,97

4,98

1,00

100

Masse volumique du mélange gazeux :

ρ = mt / V = mt / (nt RT / P).

T = 400 + 273 = 673 K ; P = 1,5 atm ;

A.N: ρ = 1,5 x 100 / (6,4019 x 0,082 x 673) = 0,425 g L-1.

Exo 3: (1) mA = 2 g ; P1 = 1 atm ; (2) mA + mB = 5 g ;

T = 298 K;

V = Cste

P2 = 1,5 atm ; T = 298 K;

V = Cste

En appliquant la loi des gaz parfaits aux deux cas de figure, on obtient: P1V = (mA / M1) x RT P2V = (mA / M1 + mB / M2) x RT

66 (P2 / P1 = 1 + (mB x M1) / (mA x M2) ⇒ M2 / M1 = (mB / mA) x P1 / (P2 - P1) A.N: M2 / M1 = (3 / 2) x 1 / (1,5 - 1) ⇒ M2/M1 = 3.

Exo 4: 1°) mair = 38,0739 – 37,9365 = 0,1374 g.

2°) nt = 1 mole ⇔ nCH4 = 0,66 mol; nC2H6 = 0,34 mol.

P = 1 atm ; T = 298 K; Mair = 29 g.mol-1.

2-a) Q = + 60 J ; W = 0 (compression isochore)

m méth + m éth = 38,0347 – 37,9365 = 0,0982 g

⇔ ∆U = Q + W = + 60 J

P = 1 atm; T = 298 K; M mél = 16 X + 30 (1 - X)

2-b) ∆U = (nCH4Cv(CH4) + nC2H6Cv(C2H6)) x ∆T

M mél = 30 - 14 X (X = fraction molaire de CH4).

⇔ ∆T = ∆U / (nCH4Cv(CH4) + nC2H6Cv(C2H6))

En appliquant la loi des gaz parfaits aux deux

Pour les gaz parfaits CP - CV = R, et donc :

cas, on obtient:

∆T = ∆U / [nCH4(Cp(CH4) – R) + nC2H6(Cp(C2H6) – R]

PV/RT =m air / 29 = (mméth + méth) / (30 - 14X)

⇒ ∆T = ∆U / (nCH4 Cp(CH4) + nC2H6 Cp(C2H6) – R)

⇔ X = [30 x m air – 29 x (m méth + m éth)]/(14 x m air) A.N: X = [30

x

0,1374



29

x

0,0982] / (14

x

0,1374) = 0,66.

A.N: ∆T = 60/(0,66 x 35 + 0,34 x 50 - 8,31)

⇒ ∆T = 1,89°. Tf = 25 + 1,89 = 26,89 °C.

Exo 5: CV = 5/2 R; CP = 7/2 R; n = 1 mole. W = - P1 x (2V1 - V1) = -P1V1. ∆UAB = Cv x (TB – TA) = 5/2 R x (TB - TA) = 5/2 x (RTB - RTA)

2P1

C

Pression

Transformation AB (isobare):

∆UAB = 5/2 x [P1(2V1) – P1V1)] = 5/2 P1V1. QAB = ∆UAB - WAB = 5/2 P1V1 + P1V1 = 7/2 P1V1. ∆HAB = CP x (TB – TA) = 7/2 R x (TB - TA) = 7/2 x (RTB - RTA)

P1

A

V1

B

Volume 2V1

∆HAB = 7/2 x [P1(2V1) – P1V1)] = 7/2 P1V1.

Constat : Pour cette transformation isobare, la chaleur échangée est égale à la variation d’enthalpie (QP = ∆H).

67 Transformation BC (isotherme) : WBC = - nRT x ln (Vf / Vi) = -nRT x ln (V1 / 2V1) = 2P1V1 x ln 2 QBC = nRT x ln (Vf / Vi) = nRT x ln (V1 / 2V1) = -2P1V1 x ln 2 ∆UBC = ∆HBC = 0 (transformation isotherme : ∆T = 0) Transformation CA (isochore) : WCA = 0 (V constant) ∆UCA = Cv x (TA – TC) = 5/2 R x (TA – TC) = 5/2 x (RTA – RTC) ∆UCA = 5/2 x [P1V1 – (2P1)V1] = -5/2 P1V1 QCA = ∆UCA - WCA = - 5/2 P1V1 + 0 = -5/2 P1V1. ∆HCA = CP x (TA – TC) = 7/2 R x (TA – TC) = 7/2 x (RTA – RTC) ∆HCA = 7/2 x [P1V1 – (2P1)V1] = -7/2 P1V1. Calcul des différentes grandeurs énergétiques pour le cycle: ΣQ = QAB + QBC + QCA = 7/2 x P1V1 -2P1V1 x ln 2 - 5/2 P1V1 = P1V1 - 2P1V1 x ln 2. ΣW = WAB + WBC + WCA = - P1V1 + 2P1V1 x ln 2 + 0 = - P1V1 + 2P1V1 x ln 2. Σ∆H = ∆HAB + ∆HBC + ∆HCA = 7/2 P1V1 - 7/2 P1V1 = 0 Σ∆U = ∆UAB + ∆UBC + ∆UCA = 5/2 P1V1 - 5/2 P1V1 = 0 Conclusion : ΣQ ≠ 0, ΣW ≠ 0: Q et W dépendent du chemin suivi; Q et W ne sont pas des fonctions d’état. Σ∆H = Σ∆U = 0 : ∆H et ∆U sont des fonctions d’état ; elles ne dépendent pas du chemin suivi mais uniquement de l’état initial et de l’état final. Si les états initial et final sont confondus comme dans un cycle, Σ∆H et Σ∆U doivent être nulles.

Exo 6: 1°) C2H4 (g) + H2 (g)

C2H6 (g)

On peut utiliser une combinaison des équations (4) et (3) : -1 x (4) : C2H4 (g)

2 C (graphite) + 2 H2 (g)

-∆H4

1 x (3) : 2 C (graphite) + 3 H2 (g) C2H6 (g) ∆H3 ---------------------------------------------------------------------------------C2H4 (g) + H2 (g) C2H6 (g) ∆H = ∆H3 - ∆H4 = -20,2 – 12,6 = -32,8 kcal ∆U = ∆H – RT ∆n A.N: ∆U = -32,8 – 2 x 298 x (-1) x 10-3 = -32,2 kcal.

68 2°) C2H4 (g) + 3 O2 (g)

2 CO2 (g) + 2 H2O (l)

C2H4 (g) + H2 (g)

∆H

C2H6 (g)

1 x (1) : C2H6 (g) + 7/2 O2 (g)

2 CO2 (g) + 3 H2O (l)

∆H1

-1 x (2) : H2O (l) H2 (g) + 1/2 O2 (g) -∆H2 ---------------------------------------------------------------------------------C2H4 (g) + 3 O2 (g) 2 CO2 (g) + 2 H2O (l) ∆HC = ∆H1 -∆H2 + ∆H A.N : ∆HC = -372,8 + 68,3 –32,8 = -337,3 kcal.

Exo 7: On peut utiliser le cycle enthalpique suivant : ∆HC CnH2n+2 (g)

+

½ (3n+1) O2 (g)

(n - 1)EC-C + (2n + 2)EC-H

0

n CO2 (g) +

n x ∆H°f CO2 (g)

(n+1) H2O (l) (n+1) x ∆H°f H2O (l)

n C (g) + 2(n+1) H (g) + ½ (3n+1) O2 (g) n C (s) + ½ (3n+1) O2 (g) + (n+1) H2 (g) [n ∆HS+ (n+1) ∆Hdiss H-H] Loi de Hess : ∆HC = - (n-1)EC-C - (2n + 2)EC-H - n ∆HS - (n + 1) ∆Hdiss H-H + n ∆H°f CO2 (g) + (n + 1) ∆H°f H2O (l) A.N: ∆HC = - (n - 1) x (-348) – (2n + 2) x (-414) – 717 x n – (n + 1) x 436 – 393,51 n – (n+1) x 285,85 ∆HC = 656,36 n – 241,85 kJ par mole d’alcane. n

1

2

3

4

5

Formule

CH4

C2H6

C3H8

C4H10

C5H12

Nom

Méthane

Éthane

Propane

Butane

Pentane

∆H combustion (kJ.mol-1)

-898,2

-1554,6

-2210,9

-2867,3

-3523,6

Exo 8: 4 HCl (g) + O2 (g)

2 H2O (g) + 2 Cl2 (g)

1°) ∆HR = 2 ∆H°f (H2O (g)) - 4 ∆H°f (HCl (g)) A.N : ∆H°298 = 2 x (-241,8) – 4 x (-92,3) = -114,4 kcal. 723

° = ∆H298 ° + 2°) ∆H723

∫ (Συ C )dT 298

i pi

(Les CP sont constants, et il n'y a pas de changements de phase).

⇒ ∆H°723 = ∆H°298 + (2 CP Cl2 (g) + 2 CP H2O (g) - CP O2 (g) - 4 CP HCl (g)) x (723 - 298) ∆H°723 = -114,4 + (723 - 298) x (2 x 37,6 + 2 x 38,4 - 31,4 - 4 x 30,1) x 10-3 = -114,315 kJ.

69

∆U°723 = ∆H°723 - RT ∆n Α.Ν: ∆U°723 = -114,315 - 8,31 x 723 x (-1) x 10-3 = -108,307 kJ. 3°) Réacteur fermé adiabatique: V = constante, pas de pertes thermiques. La chaleur de réaction à volume constant (∆U) sert à échauffer les produits de réaction et les gaz introduits dans le réacteur mais qui n’ont pas participé à la réaction (ici N2 seulement). Avant réaction Après réaction

4 HCl (g) + O2 (g) + N2 (g) 4 2 8 0 1 /

2 H2O (g) + 2 Cl2 (g) + N2 (g) 0 0 / 2 2 8

- ∆U = Σ(ni CVi) ∆T = (1 x CVO2 (g) + 2 x CVH2O (g) + 2 x CVCl2 (g) + 8 x CVN2 (g)) x (T - 723) T = (- ∆U) / (CVO2 (g) + 2 x CVH2O (g) + 2 x CVCl2 (g) + 8 x CVN2 (g)) + 723 Pour chaque gaz, CV = CP - R T = (- ∆U) / (CPO2 (g) + 2 x CPH2O (g) + 2 x CPCl2 (g) + 8 x CPN2 (g) - 13R) + 723 A.N: T = 108307 / (31,4 + 37,6 x 2 + 38,4 x 2 + 30,2 x 8 - 13 x 8,31) + 723 = 1064,7 K.

Exo 9: C6H5COOH + 15/2 O2

7 CO2 + 3 H2O

2°) La variation de température dans le

m = 0,1 g ; Q = 6300 cal/g ; ∆T = 0,9 °C

calorimètre est due au dégagement de

Cette première expérience nous permet de calculer la

chaleur provoqué par les réactions de

valeur en eau du calorimètre :

combustion du propane et du fer.

m x Q = µ cal x ∆T ⇒ µ cal = 0,1 x 6300 / 0,9 = 700 cal/°C

QT = µcal x ∆T

1°) C3H8 + 5 O2

QT = -∆H comb C3H8 x n - ∆H comb fer x mfer

3 CO2 + 4 H2O

nC3H8 = 0,44/44 = 0,01 mol; ∆T = 8°C

∆H comb = (-µ cal x ∆T – ∆H comb fer x mfer) / n

Q = µ cal x ∆T

A.N: ∆Hcomb = (-700x 8+ 1572 x0,19)/0,01

∆H = -Q / n = -µ cal x ∆T / n = -700 x 10-3 x 8 / 0,01

∆Hcomb = -530,132 kcal.mol-1.

∆H = -560 kcal.mol-1.

B-2) Série de TD n°2 (2002-2003) Exo 1: O2 : m = 10 g ; État A: PA = P0 = 1 atm; TA = T0 = 273 K; VA = V0.

1°) nombre de mole de O2 et valeur de P0V0 :

70 n = m/M = 10/32 = 0,3125 ; P0V0 = nRT0 = 0,3125 x 0,082 x 273 # 7 L atm = 708,83 J. Puisque P0 = 1 atm, V0 = 7 L.

2°) Transformation isochore A→ B : PB = 2PA = 2P0 = 2 atm ;

2,64

D

2

B

VB = VA = V0 = 7 L ; TB = ? constant ; on a donc PB/TB = PA/TA ⇒ TB = PB x TA / PA = 2 TA = 546 K.

3°) Transformation isobare A→ C :

P (atm)

Pour une transformation isochore, le rapport P/T est

VC = 2VA = 2V0 = 14 L ; PC = PA = P0 = 1 atm ; TC = ? Pour une transformation isobare, le rapport V/T est

1 A 7 Volume (L)

C 14

constant ; on a donc VC/TC = VA/TA ⇒ TC = VC x TA / VA = 2 TA = 546 K

4°) ∆UAC = n x CV x (TC-TA) = n x CV x TA = 5/2 x (nRTA) = 5/2 x P0V0 = 1772,08 J. ∆UAB = n x CV x (TB-TA) = n x CV x TA = 5/2 x (nRTA) = 5/2 x P0V0 = 1772,08 J. ∆HAC = n x CP x (TC-TA) = n x CP x TA = 7/2 x (nRTA) = 7/2 x P0V0 = 2480,91 J. ∆HAB = n x CP x (TB-TA) = n x CP x TA = 7/2 x (nRTA) = 7/2 x P0V0 = 2480,91 J. 5°) Etat D : VD = VA =7 L ; TD = 721 K ;

⇒ PD = nRTD/VD = 0,3125 x 0,082 x 721/7 = 2,64 atm. Le diagramme de Clapeyron est montré sur la figure ci-dessus. Le travail échangé au cours du cycle BDCB est la somme des travaux échangés pendant les différentes transformations : W BDCB = W BD + W DC + W CB BD transformation isochore : W BD = 0. DC transformation adiabatique : Q DC = 0 ;

⇒ W DC = ∆U DC = n x CV x (TC-TD) CB transformation isotherme : W CB = -nRTC x ln(VB/VC) ;

71

⇒ W BDCB = n x CV x (TC-TD) + nRTC x ln (VC/VB). A.N: W BDCB = 0,3125 x 8,314 x [2,5 x (546 - 721) + 546 x ln (14/7)] = - 153,4 J. Remarque : Le travail échangé au cours du cycle n’est pas nul car le travail n’est pas une fonction d’état.

Exo 2: Pb (s) + ½ O2 (g) → PbO (s)

∆H1 = ∆H°f PbO (s) = -219 kJ;

Pb3O4 (s) → 3 PbO (s) + ½ O2 (g)

∆H2 = 3 ∆H°f PbO (s) - ∆H°f Pb3O4 (s) = + 76,9 kJ;

Pb3O4 (s) + O2 (g) → 3 PbO2 (g)

∆H3 = 3 ∆H°f PbO2 (s) - ∆H°f Pb3O4 (s) = -94,9 kJ.

La résolution de ce système donne :

∆H°f PbO2 (s) = (∆H3 - ∆H2 + 3 ∆H1)/3 ; A.N: ∆H°f PbO2 (s) = (-94,9 – 76,9 – 3 x 219) / 3 -276,3 kJ.

Exo 3: Cdiamant → Cgraphite

∆HR = ∆H°f Cgraphite – ∆H°f Cdiamant

Cdiamant + O2 (g) → CO2 (g)

∆H1 = ∆H°f CO2 (g) – ∆H°f Cdiamant

Cgraphite + O2 (g) → CO2 (g)

∆H2 = ∆H°f CO2 (g) – ∆H°f Cgraphite

∆HR = ∆H1 – ∆H2 = -94,48 + 94,03 = -0,45 kcal.

Exo 4: Q = m x c x ∆T ⇒ c = Q / (m ∆T) A.N: c = 112 / (100 x 10) = 0,112 cal g-1 °C-1.

Exo 5: a) Enthalpie standard de formation de la pyridine, en phase gazeuse à 298 K : C5H5N (l) + 25/4 O2 (g)

5CO2 (g) + 5/2 H2O (l) + 1/2 N2 (g)

∆HR = -2779,5 kJ.

∆HR = 5 x ∆H0f CO2 (g) + 2,5 x ∆H0f H2O (l) - ∆H0f C5H5N (l); ∆H0vap C5H5N = ∆H0f C5H5N (g) - ∆H0f C5H5N (l) ⇒ ∆H0f C5H5N (g) = ∆H0f C5H5N (l) + ∆H0vap C5H5N; ∆H0f C5H5N (g) = - ∆HR + 5 x ∆H0f CO2 (g) + 2,5 x ∆H0f H2O (l) + ∆H0vap C5H5N A.N: ∆H0f C5H5N (g) = 2779,5 – 5 x 393,1 – 2,5 x 285,6 + 40,4 = 140,4 kJ.

72 H

b)

ΣEl

5 C (g) + 5 H (g) + N (g)

H C C

H C

N 5E 0,

H 5E 2,

≡N

ub

-H

Ls -5

0

5 C (s) + 5/2 H2 (g) + 1/2 N2 (g)

H

C f

(g) 5N H 5

N C

H

C H

D’après la loi de Hess :

Σ El = 2 EC=C + EC-N + EC=N + 2 EC-C + 5 EC-H = -5 Lsub C(s) + 2,5 EH-H + 0,5 EN≡N + ∆H0f C5H5N (g);

⇒ ∆H0f C5H5N (g) = 2 EC=C + EC-N + EC=N + 2 EC-C + 5 EC-H + 5 Lsub C(s) - 2,5 EH-H - 0,5 EN≡N A.N : ∆H0f C5H5N (g) = -614,4 x 2 – 291,3 – 614,4 – 2 x 347,3 – 5 x 413 + 5 x 716,7 + 2,5 x 435,5 + 945,5/2

∆H0f C5H5N (g) = 250,9 kJ. c) La valeur obtenue avec les énergies de liaison est plus élevée que celle obtenue avec l’enthalpie de combustion. Ceci est dû au fait que les liaisons dans la molécule de pyridine sont délocalisées à cause des phénomènes de résonance :

H

H

H C C

H

C C N

H C

C

H

H

H

C C

N C

H

H

H C C

H C

C

N C

H

H

C H

La valeur 140,4 – 250,9 = - 110,5 kJ correspond à l’énergie de résonance. Exo 6: T0 = 29 8 K

∆ H°1

T1 = 67 3 K

∆H °2 9 8

H 2 ( g ) + 1 / 2 O 2 (g )

∆H °4

H 2 O (l)

H 2 O (l)

∆H°2

T = 37 3 K

−∆ H ° v a p ∆H °6 7 3

1

H 2 ( g ) + / 2 O 2 (g )

H 2 O (g) T = 373 K ∆H °3

H 2 O (g )

Loi de Hess : ∆H°298 = ∆H°1 +∆H°2 +∆H°673 +∆H°3 - ∆H°vap +∆H°4 ∆H°673 = ∆H°vap + ∆H°298 - ∆H°1 - ∆H°2 - ∆H°3 - ∆H°4



673

∆H°1 = CP H2 (g) dT ; 298



673

∆H°2 = 1/2CP O2 (g) dT ; 298



373

∆H°3 = CP H2O (g) dT ; 673

∆H°1 = 29,97 x (673-298) + 0,5 x 4,18 x 10-3 x (6732-2982) = 12000 J; ∆H°2 = ½ x [28,26 x (673-298) + 0,5 x 2,53.10-3 x (6732-2982)] = 5529 J;



373

∆H°4 = CP H2O (g) dT 673

73

∆H°3 = 30,01 x (373-673) + 0,5 x 10,71 x 10-3 x (3732-6732) = - 10683 J; ∆H°4 = 75,47 x (298-373) = -5660 J. A.N: ∆H°673 = 40700 – 285600 – 12000 – 5529 + 10683 + 5660 = -246086 J. Exo 7:

Masse de solution : m = Vsol x ρsol = (125 + 125) x 1 = 250 g ; Valeur en eau du calorimètre : µ = 10 g ; Elévation de température : ∆θ = 17,3 – 14 = 3,3°C : Chaleur reçue par l’ensemble calorimètre + solutions : Q = (m + µ) x c x ∆θ c = chaleur spécifique de l’eau = 1 cal g-1 (°C)-1, ou 4,18 J g-1 (°C)-1. La quantité de chaleur Q reçue par le calorimètre est libérée par la réaction de neutralisation de 125 mL de HCl 0,5 M, soit par 0,0625 mol d’acide. La réaction de neutralisation est donc exothermique, et la chaleur de neutralisation est, par mole d’acide :

∆U = ∆H = -(m + µ) x c x ∆θ/0,0625. A.N: ∆H = -(250 + 10) x 4,18 x 3,3/0,0625 = -57,38 kJ mol-1. Remarque : Cette valeur peut représenter la chaleur de neutralisation d’un monoacide fort quelconque par une monobase forte quelconque car la neutralisation se résume à la réaction : H3O+ + OH- → 2H2O

C-1) Série de TD n° 3 (2001-2002) Exo 1:

PCl5 (g) n0 n0 - x n0 (1 - α)

PCl3 (g) + Cl2 (g) 0 0 x x n0 α n0 α

1-α α α 1+α 1+α 1+α 1°) d = M / 29 = Σ (Xi Mi) /29

d = XPCl5 x MPCl5 + XPCl3 x MPCl3 + XCl2 x MCl2 d = [(1 - α) x 208,5 + 137,5 x α + 71 x α] / (1 + α) / 29 d = 208,5 / (1 + α) / 29 ⇒ α = 208,5 / 29d - 1 A.N: α = 208,5 / (29 x 4,62) - 1 = 0,556. 2°) KP = PPCl3 x PCl2 / PPCl5 = (XPCl3 x XCl2 / XPCl5) x P = P x α² / (1 - α²) A.N : KP = 1 x 0,556² / (1 - 0,556²) = 0,448 atm.

3°) La température étant constante, KP reste constant. KP = P’ x α’² / (1 - α’²) ⇒ α’ = [KP / (P’ + KP)]0,5 A.N: α’ = [0,448 / (0,2 + 0,448)]0,5 = 0,831.

74 Ce résultat pouvait être prévu avec le Principe de Le Châtelier : la diminution de la pression provoque un déplacement de l’équilibre dans le sens d’une augmentation du nombre de moles de gaz, c’est-à-dire, dans ce cas précis, vers une plus grande dissociation de PCl5. Exo 2:

N2O4 (g) n0 n0 - x n0 (1 - α) 1-α 1+α

A.N: KP = 0,05 atm ; P1 = 1 atm ; P2 = 0,5 atm :

2 NO2 (g) 0 2x 2n0 α 2α 1+α

α1 = [0,05 / (4 + 0,05)]0,5 = 0,111. α2 = [0,05 / (2 + 0,05)]0,5 = 0,156. Si la pression diminue de moitié, α augmente de 0,045

a) KP = PNO2² / PN2O4 = 4α²P / (1 - α²)

α = [KP / (4 P + KP)]0,5

b) α = [KP / (4 P + KP)]0,5 α² = KP / (4 P + KP)

P = KP (1- α²) / 4α²

α ≤ 0,01 ⇒ P ≥ 0,05 x (1 - 0,0001) / 0,0004

α1 = [KP / (4 P1 + KP)]0,5 ;

⇒ P ≥ 124,9875

α2 = [KP / (4 P2 + KP)]0,5

Pression minimale ≅ 125 atm.

Exo 3:

CO2 (g) CO (g) + ½ O2 (g) n0 0 0 n0 - x x x n0 (1 - α) n0 α n0 α/2 1-α α α 1 + α/2 1 + α/2 1 + α/2 3/2 KP = α / (1 - α) / (2 + α)0,5 x P 0,5 ⇔ KP² = [α / (1 - α)² / (2 + α) ] x P 3

KP = 1,45 x 10 10-3 ⇒ la formule donnant le pH d’une solution d’acide faible n’est pas applicable :

93 HCOO- + H3O+ 0 0 x x

HCOOH + H2O CA CA – x

KA = x2 / (CA - x) ⇔ x2 + x KA – KACA = 0 La résolution de cette équation du second degré donne: x = {-KA + (KA2 + 4 KAC)0,5} / 2 = {-1,77 x 10-4 + ((1,77 x 10-4)2 + 4 x 1,77 x 10-4 x 0,01)0,5} / 2 x = [H3O+] = 0,00124 ⇒ pH = -log (0,00124) = 2,90.

1-2-1) pH du mélange de 100 mL de A et 25 mL de C: Le nombre de moles initial de A est n0A = CAVA = 0,1 x 0,1 = 0,01 mol Le nombre de moles initial de C est n0C = CCVC = 0,1 x 0,025 = 0,0025 mol La réaction entre la base NH4OH et l’acide HCl est totale ; le bilan s’établit donc comme suit : NH4OH 0,01 0,0075

+

HCl 0,0025 0

NH4Cl + H2O ≈0 0,0025

A la fin de la réaction on a un mélange de la base faible NH4OH et de son acide conjugué NH4+, c’est-à-dire une solution tampon ; pH = pKA + log ([NH4OH] / [NH4+]) = 14 – pKB + log (n NH4OH / n NH4+) A.N: pH = 14 + log (1,79 x 10-5) + log (0,0075 / 0,0025) = 9,73.

1-2-1) pH du mélange de 10 mL de A et 100 mL de D: La réaction entre la base NH4OH et l’acide HCOOH est totale ; on a donc le bilan suivant : NH4OH 0,001 0

+

NH4+ + ≈0 0,001

HCOOH 0,001 0

HCOO≈0 0,001

+

H2O

A la fin de la neutralisation, on a un sel d’acide et de base faibles ; le pH est donc la demi-somme des pKA des deux couples acide-base : pH = ½ [pKA(NH4+) + pKA(HCOOH)] = ½ [-log (1,77 x 10-4) + 14 + log (1,79 x 10-5)] = 6,50. Exo 2:

2-2) Réaction globale :

2-1) Schéma de la pile : e-

Anode



Al

i

Cathode Cu

+

Al 3+ + 3 e−

Anode: Al Cathode : Cu

2+

+ 2 e−

Cu x 3

Réaction globale: 2 Al + 3 Cu 2+ Al2(SO4)3 0,1M

CuSO4 0,1 M

paroi poreuse

x2 2 Al 3+ + 3 Cu

94

2-3) Constante d’équilibre : log K = n1 x n2 (E°ox – E°réd) / 0,06 log K = 2 x 3 x (0,34 + 1,66)/0,06 = 200 ⇒ K = 10200.

2-4) Concentrations finales des ions lorsque la pile est usée: Lorsque la pile est usée, on considère que la réaction est totale ; Cu2+ disparaît : 2 Al + 3 Cu 2+ t=0

/

0,1

t∞

/

≈0

2 Al 3+ + 3 Cu 0,2

Compartiment cathodique : [Cu2+] = 0

/

Compartiment anodique :

0,2 + 0,1 x 2/3 /

[Al3+] = 0,267 M ; [SO42-] = 0,4 M

D-7) Corrigé du partiel d’avril 2004 Exo 1 : mg = 36 g Cg = 38 J mol-1 K-1 θ1 = -10 °C LF = 6000 J mol-1.

glace

me = 90 g Ce = 76 J mol-1 K-1 θ2 = 50 °C

+ eau

=

eau : θF = ?

1-1) Soient QR et QC, les quantités de chaleur reçue par la glace et cédée par l’eau : QR = mg x [Cg x (0 - θ1) + LF + Ce x (θF - 0)] / 18; QC = me x [Ce x (θF - θ2)] /18 ; Puisqu’il n’y a pas d’échanges de chaleur, QR = QC. mg x [Cg x (0 - θ1) + LF + Ce x (θF - 0)] / 18 = me x [Ce x (θ2 - θF)] /18 ⇒ θF = [(me x Ce x θ2 + mg x (Cg x θ1 - LF)] / [Ce x (me + mg)]

A.N : θF = [90 x 76 x 50 + 36 x (38 x (–10) - 6000)] / [76 x (60 + 36)] = 11,7 °C. 1-2) mT = 126 g Q 1 = n T x Ce x (θvap − θi) H2O(l) (θi = 1,7 0°C)

Q 3 = n T x Ce x (θf − θi)

Q2 = n T x LV H2O(l)

(θvap =100°C)

H2O(g)

H2O(g)

(θvap =100°C)

(θf = 150 °C)

QT = Q1 + Q2 + Q3 = (126/18) x [76 x (100 – 11,7) + 40700 + 34 x (150 – 100)] = 343775,6 J Exo 2 : Soit x la quantité de PCl5 formé ; on obtient le bilan : PCl5(g)

t=0 0,5 téq 0,5 − x

PCl3(g) + Cl2(g) 0 0,5 x 0,5 + x

Σ 1 1+x

95

-

XPCl3 = x / (1 + x) ; XCl2 = (0,5 + x) / (1 + x) ; XPCl5 = (0,5 − x) / (1 + x) ;

PT = nT x RT/V = (1 + x) x RT/V KP =

XCl2 x XPCl3 XPCl5

x PT

=

x (0,5 + x)

RT x

(0,5 - x)

V

2 ⇒ x + x (0;5 + KPV/RT) - 0,5 KPV/RT = 0

⇒ x = {-(0,5 + 1,7 x 2 / 0,082 / 523) + [(0,5 + 1,7 x 2 / 0,082 / 523)2 + 4 x 0,5 x 1,7 x 2 / 0,082 / 523]0,5}/2

x = 0,0618 ⇒ X PCl3 = 0,058 ; XCl2 = 0,529 ; XPCl5 = 0,413

-

PT = (1 + x) x RT/V = 1,0618 x 0,082 x 523 /2 = 22,8 atm.

-

d = Mmél/MCl2 = [XPCl3.MPCl3 + XCl2.MCl2 + XPCl5.MPCl5] / MCl2

A.N: d = [0,058 x 137,5 + 0,529 x 71 + 0,413 x 208,5] / 71 = 1,854.

D-8) Corrigé de l’examen écrit de juillet 2004 Exo 1 : 1°) C6H6 (l) + 15/2 O2 (g) → 6 CO2 (g) + 3 H2O (l) 2°) ∆UR = ∆HR – RT x ∆n = ∆HR – RT x (6-15/2) Combustion à P constante ⇒ ∆HR = -Q/n = (M/m) x (-Q) ∆U = (M/m) x (-Q) +1,5RT A.N : ∆U = -3265 x 78/78 + 1,5 x 8,31 x 298 x 10-3 = -3261,3 kJ. 3°) Si le benzène est liquide à 80 °C, il n’y a aucun changement de phase ; comme les Cp sont constants, on peut écrire, d’après la loi de Kirchhoff : ∆H353 = ∆H298 + (353-298) x (6 Cp (CO2 (g)) + 3 Cp (H2O (l)) - Cp (C6H6 (l)) – 7,5 Cp (O2 (g)) A.N : ∆H353 = -3265 + 55 x (6 x 29 + 3 x 76 – 148 – 7,5 x 29) x 10-3 = -3263 kJ. Si le benzène est gazeux, on a : ∆Hcomb C6H6 (g) = ∆Hcomb C6H6 (l) - ∆Hvap C6H6 (g) A.N: ∆Hcomb C6H6 (g) = -3262,99 – 30,84 = -3293,83 kJ. 4°) QV = -(m/M) x ∆U = -(m/M) x (∆H –RT x ∆n)

A.N: QV = (-3,9/78) x (-3293,83 – (6-7,5-1) x 8,314 x 353 x 10-3) = 164,32 kJ. Exo 2 : 1°) La valeur du pKA de NaNO2 indique une base faiblement ou moyennement dissociée. KB/C = 10-14+3,4/(2,76/69) = PT =

2 T

α x (2-α) 2

(1-α)

0,5

x KP

AN : PT = (0,99 x 1,01/(0,012 x 2,51 x 104))0,5 = 0,63 atm.

3°) On applique la loi de Van’t Hoff : KP2 = KP1 x e(∆H x (1/T1-1/T2)/R); A.N: KP2 = 25100 x e(−90960 x (1/298-1/473)/8,314) = 3,166 x 10-2 atm-2.

4°)

KP2 =

α' x (2-α' ) 2

(1-α' )

xP

2 T

> KP

2

x PT2

=

α' x (2-α' ) (1-α' )2

α' 2 - 2 α' + (KP2 x PT2 / (1+ KP2 x PT2) = 0 ⇔ α' = 1 - [1 - KP2 x PT2 / (1+ KP2 x PT2)]0,5 Α.Ν: Α.Ν α' = 1 - [1 – 0,03166 x 0,632 / (1 + 0,03166 x 0,632)]0,5 = 0,0062. PT' 2 =

5°)

α' x (2-α' ) (1-α' )2 x KP 2

AN: PT' = [(0,99 x 1,01/0,012/0,03166]0,5 = 562 atm.

6°) T = 473 K

P1 = 0,63 atm ⇒ α1 = 0,0062 L’augmentation de la pression déplace l’équilibre dans le sens de la formation de CH3OH, c-à-d, dans notre P2 = 562 atm ⇒ α2 = 0,99 cas, vers la diminution du nombre de moles de gaz. T1 = 298 K ⇒ α1 = 0,99

L’augmentation de la température déplace l’équilibre PT = 0,63 atm dans le sens de la formation de CO, c’est-à-dire, dans T2 = 473 K ⇒ α1 = 0,006 le cas présent, dans le sens endothermique Les variations du taux de conversion avec la pression et avec la température sont conformes au principe de le Châtelier. Exo 2 :

1°) Nous avons deux acides faibles. Calculons les rapports KA/C : Solution A : HCOOH, C1 = 0,4 M ; et KA1 = 1,8 x 10-4 ; KA1/C1 = 1,8 x 10-4 / 0,4 = 4,5 x 10-4 < 10-3 Solution B : CH3COOH, C2 = 0,4 M et KA2 = 1,8 x 10-5 ; KA2/C2 = 1,8 x 10-5 / 0,4 = 4,5 x 10-5 < 10-3. KA1/C1 et KA2/C2 étant inférieurs à 10-3, on peut appliquer la formule du pH d’un acide faible : pH = ½ (pKA –log C) = - ½ log(KAC) : A.N: sol A : pH = - - ½ log(1,8x 10-4 x 0,4) = 2,07.

sol B : pH = - - ½ log(1,8x 10-5 x 0,4) = 2,57. 2°) Soient C1’ et C2’ les nouvelles concentrations molaires de HCOOH et CH3COOH dans le mélange ;

99 C1’ = C1V1/VT = 0,4 x 100/200 = 0,2 M ;

C2’ = C2V2/VT = 0,4 x 100/200 = 0,2 M

KA1/C1’ = 1,8 x 10-4 / 0,2 = 9 x 10-4 < 10-3 ;

KA2/C2 = 1,8 x 10-5 / 0,2 = 9 x 10-5 < 10-3

Les acides formique et acétique restent très peu dissociées, et donc la formule donnant le pH d’un mélange d’acides faibles est applicable : A.N: pH = - ½ log (KA1C1’ + KA2C2’) = - ½ log (1,8 x 10-4 x 0,2 + 1,8 x 10-5 x 0,2) = 2,20.

3°) NaOH (n = 3,2/40 = 0,08 mol) réagit avec les acides (n0 = 0,2 x 0,2 =0,04 mol) suivant le bilan réactionnel : ti tf

2NaOH + HCOOH + CH3COOH → HCOONa + CH3COONa + 2 H2O 0,08 0,04 0,04 0 0 0 0 0 0,04 0,04

(mol) (mol)

La solution finale est un mélange de deux sels d’acide faible et de base forte, c’est à dire de deux bases faibles (HCOO- et CH3COO-). Calculons les rapports Kb/C : HCOO- : Kb1/Cb1 = (10-14/1,8 x 10-4)/0,2 = 2,78 10-10 < 10-3 ; CH3COO- : Kb2/Cb2 = (10-14/1,8 x 10-5)/0,2 = 2,78 10-9 < 10-3 ; Les rapports Kb/C étant inférieurs à 10-3, on peut appliquer la formule du pH d’un mélange de deux bases faibles : pH = 7 + ½ log (Cb1/KA1 + Cb2/KA2) A.N: pH = 7 + ½ log (0,2/1,8 x 10-4 + 0,2/1,8 x 10-5) = 9,04.

4°) Si on ajoute seulement 1,6 g de NaOH, les acides seront en en excès par rapport à NaOH. On appellera x le nombre de mol de NaOH qui réagit avec HCOOH et y le nombre de mol de NaOH qui réagit avec CH3COOH : ti tf

2NaOH + HCOOH + CH3COOH → HCOO- + CH3COO- + 2Na+ + 2 H2O 0,04 0,04 0,04 0 0 (mol) #0 0,04-x 0,04-y x y 0,04 (mol)

Les lois d’action de masse devant être vérifiées pour les équilibres acido-basiques, il s’ensuit : KA1 = [HCOO-] x [H3O+] / [HCOOH] = [H3O+].x / (0,04 - x) KA2 = [CH3COO-] x [H3O+] / [CH3COOH] = [H3O+].y / (0,04 - y) Les réactions de neutralisation entre NaOH et les acides sont quasi-totales. Par conséquent NaOH disparaît x + y = 0,04. On a donc :

KA1 = [H3O+].x / y ;

KA2 = [H3O+].y / x

KA1 x KA2 = [H3O+]2 ⇒ pH = - ½ log (KA1 x KA2) A.N: pH = - ½ log (1,8 x 10-4 x 1,8 x 10-5) = 4,24.

KA1 = [H3O+].x / (0,04 - x) ⇒ x = 0,04 x KA1/([H3O+] + KA1) = 0,04 x KA1/((KA1 x KA2)0,5 + KA1).

100 KA2 = [H3O+].y / (0,04 - y) ⇒ y = 0,04 x KA2/([H3O+] + KA2) = 0,04 x KA2/((KA1 x KA2)0,5 + KA2). [HCOO-] = x /VT = 0,04 x KA1/((KA1 x KA2)0,5 + KA1)/VT. [CH3COO-] = y /VT = 0,04 x KA2/((KA1 x KA2)0,5 + KA2)/VT. A.N: [HCOO-] = 0,04 x 1,8 x 10-4/((1,8 x 10-4 x 1,8 x 10-5)0,5 +1,8 x 10-4))/0,2 = 0,152 M.

[CH3COO-] = 0,04 x 1,8 x 10-5/((1,8 x 10-4 x 1,8 x 10-5)0,5 +1,8 x 10-5))/0,2 = 0,048 M.

101 Bibliographie sommaire

1°) Chimie Physique ; Cours Et Exercices Corriges ; 4e Edition, Dunod - 11/02/1998

2°) Paul ARNAUD, Chimie Physique ; Exercices Résolus, Dunod - 22/10/1998

3°) Michel Bertin, Thermodynamique ; Bordas - 08/03/1999

4°) Paul Roux, Thermodynamique ; 1e Année ; Mpsi Pcsi Ptsi ; Ellipses Marketing - 08/04/1998

5°) Alain Gruger, Thermodynamique Et Equilibres Chimiques - Cours Et Exercices Résolus ; Dunod - 15/04/1997

6°) René Didier, Chimie Générale ; Editions Jb Baillière - 01/01/1981

7°) François Duparc, Chimie Générale, Aide Mémoire ; Dunod - 10/11/1997

8°) René Didier, Chimie Générale : Cours Et Exercices d'Application ; Tec Et Doc - 01/04/1997.